rmm 8.pdf

64
CUPRINS - 0 - Cuvânt înainte din partea colectivului redacţional _________________ 1 Membrii Filialei Mehedinţi a S.S.M.R. ____________________________ 2 Concursul Interjudeţean de Matematică “Petre Sergescu” Subiecte_________________________________________________________ 3 Premianţi ________________________________________________________ 6 Simpozionul de referate _____________________________________________ 7 Note matematice Pregatire pentru olimpiade şi concursuri _________________________ 10 Probleme pentru Olimpiada ______________________________________ 11 Asupra Numerelor Algebrice _____________________________________ 14 Teme pentru grupele de performanţă Clasa a V-a Operaţii cu numere naturale binare ______________________________ 16 Clasa a V-a Selectie de probleme __________________________________________ 18 Clasa a VII-a Operaţii cu numere iraţionale _____________________________________ 20 Clasa a VIII-a Principiul Invariantului __________________________________________ 23 Clasa a IX-a Invarianţi ________________________________________________________ 29 Clasa a X-a Câteva probleme generate de exponenţiala de bază 2 ______________ 27 Clasa a XII-a Centralizatorul unui element într-un grup. Centrul unui grup. Teorema lui Lagrange. Indicele unui subgrup într-un grup _________ 29 Clasa a XII-a Asupra elementelor simetrizabile dintr-un monoid _________________ 32 Clasa a XII-a Extinderi de inele şi corpuri _______________________________________ 34 Probleme propuse ________________________________________________ 36 Subiecte Olimpiada de matematică etapa locală 2008 _____________ 45 Teste de verificare – admitere clasa a V-a _________________________ 48 Premianţii concursurilor din 2007 ______________________________ 52 Rubrica rezolvitorilor ________________________________________________ 60

Upload: silviu-boga

Post on 15-Jan-2016

225 views

Category:

Documents


14 download

TRANSCRIPT

Page 1: RMM 8.pdf

CUPRINS

- 0 -

Cuvânt înainte din partea colectivului redacţional _________________ 1 Membrii Filialei Mehedinţi a S.S.M.R. ____________________________ 2

Concursul Interjudeţean de Matematică “Petre Sergescu”

Subiecte_________________________________________________________ 3 Premianţi ________________________________________________________ 6 Simpozionul de referate _____________________________________________ 7

Note matematice

Pregatire pentru olimpiade şi concursuri _________________________ 10 Probleme pentru Olimpiada ______________________________________ 11 Asupra Numerelor Algebrice _____________________________________ 14

Teme pentru grupele de performanţă Clasa a V-a

Operaţii cu numere naturale binare ______________________________ 16 Clasa a V-a Selectie de probleme __________________________________________ 18 Clasa a VII-a Operaţii cu numere iraţionale _____________________________________ 20 Clasa a VIII-a Principiul Invariantului __________________________________________ 23 Clasa a IX-a Invarianţi ________________________________________________________ 29 Clasa a X-a Câteva probleme generate de exponenţiala de bază 2 ______________ 27 Clasa a XII-a Centralizatorul unui element într-un grup. Centrul unui grup. Teorema lui Lagrange. Indicele unui subgrup într-un grup _________ 29 Clasa a XII-a Asupra elementelor simetrizabile dintr-un monoid _________________ 32 Clasa a XII-a Extinderi de inele şi corpuri _______________________________________ 34 Probleme propuse ________________________________________________ 36 Subiecte Olimpiada de matematică etapa locală 2008 _____________ 45 Teste de verificare – admitere clasa a V-a _________________________ 48 Premianţii concursurilor din 2007 ______________________________ 52 Rubrica rezolvitorilor ________________________________________________ 60

Page 2: RMM 8.pdf

EDITORIAL

H SSM

- 1 -

Cuvânt înainte din partea colectivului

redacţional

Daca aceste randuri au ajuns in fata dumneavoastra inseamna ca am reusit si in acest an sa realizam Revista Mehedinteana de Matematica.Publicatia noastra ajunge astfel la nr.8 si va reaminteste ca scopul ei,izvorat din Statutul Societatii de Stiinte Matematice din Romania este de a contribui la ridicarea nivelului de pregatire matematica a elevilor si profesorilor mehedinteni si nu numai. In paginile revistei veti gasi reflectate o parte din activitatile Filialei Mehedinti a SSMR, lista membrilor filialei noastre la 31.12.2007(revista se concepe in perioada septembrie-noiembrie 2008 si deci asteptam pana la sfarsitul anului ca toti iubitorii de matematica sa ni se alature in continuare prin plata cotizatiei pe 2008),asa cum ne-am obisnuit deja Teme pentru grupele de performanta,problemele Editiei a IV-a a Concursului Interjudetean Petre Sergescu (cu bucurie semnalam ca sora noastra mai mare gazeta Matematica publica in Nr.9/2008 un material care reflecta concursul nostrum),probleme propuse ,lista rezolvitorilor care au trimis probleme rezolvate prin intermediul colegilor nostri entuziasti de la scolile mehedintene,si nu in ultimul rand o reflectare speram noi cat mai fidela a rezultatelor obtinute de elevii nostri la concursurile interjudetene,nationale si internationale la care au participat. Problemele invatamantului sunt multe ,dar noi cei care lucram cel putin pe teritoriul privilegiat al matematicii nu putem accepta in niciun caz concluzii semidocte afisate pe la diverse posturi de televiziune ca profesorii romani au o slaba pregatire. Credem ca si cu acesta revista putem arata oricui pe mapamond ca pregatirea dascalilor nostri este profunda si ca rezultatele lor sunt meritorii. Noi, un grup mare de profesori de matematica, suntem convinsi ca elevii nostrii au valoare,sunt bine instruiti si pot concura oricand cu colegii lor din restul lumii. Noi credem ca pregatirea matematica a tuturor elevilor si nu numai a olimpicilor este extrem de utila pentru viata lor profesionala ulterioara ,iar elevii nostri care acum lucreaza in diverse colturi ale globului in diverse profesii ne confirma ca nu ne inselam. Pentru ca RMM 8, va apare in apropierea sfarsitului de an dorim tuturor colaboritorilor nostri “La multi ani”.

Presedintele Filialei Mehedinti a SSMR,

Profesor doctor Gh.Cainiceanu

Page 3: RMM 8.pdf

EDITORIAL

- 2 -

Membrii Filialei Mehedinţi a S.S.M.R.

- 31.XII.2007 –

1 Cainiceanu Gheorghe CNT PRESEDINTE 2 Prajea Manuela CNT Vicepresedinte 3 Stretcu Daniel Colegiul.Gh.Titeica Secretar 4 Grecu Vasile Colegiul Economic Casier 5 Nănuţi Dan CNT – ISJ Membru comitet 6 Nedeianu Dan Lic. Dl. Tudor Membru comitet 7 Săceanu Victor Şc. Gen. Nr.11 Membru comitet 8 Ungureanu Octavian Colegiul.Gh.Titeica Membru comitet 9 Gimoiu Iuliana CNT CENZOR

10 Giugiuc C-tin C. Dl. Tudor CENZOR

1 Istodor Oana Lic.S. Cioculescu 41 Pit Rada Marica Gen.5 2 Antonie Rodica CNT 42 Lupu Adrian Decebal 3 Giugiuc Leonard CNT 43 Nitoiu Angela Decebal 4 Popescu Eleodor CNT 44 Lugoj Tanta Decebal 5 Paponiu Dana CNT 45 Oprita Manuela Decebal 6 Crisan Livia Halanga 46 Bejenaru Laviniu CNT 7 Ionescu Adela C.U.Drobeta 47 Croitoru Ion CE 8 Fritea Eugen C. Dl. Tudor 48 Bizdoaca Claudia CE 9 Chilea Ion C. Dl. Tudor 49 Grecu Adela CE

10 Vasilcanu Florentina C. Dl. Tudor 50 Ladaru Damiela CE 11 Cristel Ecaterina C. Dl. Tudor 51 Stoican Victor Baia 12 Ciuca Ionel C. Dl. Tudor 52 Paulescu Petre Baia 13 Badescu Emilia C. Dl. Tudor 53 Barbulescu Marin Baia 14 Moclea Adriana Gen.11 54 Simionescu Loredana Baia 15 Lapadat Petruta Gen.7 55 Falon Florica Marina 16 Capraru Dorel Gen.7 56 Petrache Elena Marina 17 Pop Veronica Severinesti 57 Bogdan Daniel Marina 18 Pandioniu Aristita Hinova 58 Deris Antoaneta Lic.4 19 Diaconescu Emilia Gura Vaii 59 Varzaru Mariana Lic.Auto 20 Bondoc Gabriela Lic.Auto 60 Vaduva Ion Liv.Paulian 21 Pasov Nicoleta Lic.Auto 61 Untaru Ilie Odoblrja 22 Bondoc Lucian Tribunal 62 Popescu Marcel Odobleja 23 Grecu Luminita C.U.Drobeta 63 Ticusi Ovidiu Odobleja 24 Calafeteanu Gh Gen.9 64 Presneanu Doru Odobleja 25 Florescu Violeta Gen.4 65 Baloi Valeria Odobleja 26 Cărbunaru Dumitru Gen.4 66 Balu Nicoleta Odobleja 27 Ianasi Ion Gen.4 67 Balasoiu Daniela Lic.S. Cioculescu 28 Mălineanu Gabriela Grn.4 68 Raducan Emilia Decebal 29 Sitaru Dan CE 69 Pupaza Ecaterina Decebal 30 Blidaru C-Tin Malovat 70 Farago Alexandru T.Lalescu 31 Fluerasu Anghel Izv.Barzii 71 Bobic Nicolae T.Lalescu 32 Budanescu Lidia Sc.1 72 Gorun Sanda T.Lalescu 33 Stioiu Petre Bistrita 73 Drula Ileana Strehaia 34 Marin Felicia Sc.9 74 Juganaru Ion Strehaia 35 Zaharia Marius Ilovat 75 Neamtu Elian CE 36 Saftoiu Gheorghe Sc.15 76 Dragotescu Alexa Sc.7 37 Vasile Tomita C.U. 77 Tomoescu Loredana Sc.7 38 Bala Dumitru C.U. 78 Patrascoiu Constantin C.U. 39 Stuparu Dragos C.U. 79 Ionica Constantin Gen.14 40 Pit-Rada Ionel Vasile CNT

Page 4: RMM 8.pdf

Petre Sergescu

H

- 3 -

SSM Concursul Interjudeţean de Matematică

“PETRE SERGESCU” Ediţia a IV-a, martie 2008, Drobeta Turnu-Severin

Prof.dr.Prajea Manuela ,Director Colegiul National Traian Prof.dr.Cainiceanu Gheorghe,Presedintele Filialei Mehedinti a SSMR

Concursul Interjudetean de matematica “Petre Sergescu” editia a IV-a, s-a desfasurat la Drobeta Turnu-Severin pe 21.03.2008. Organizatorii concursului, ca si la celelalte trei editii au fost Colegiul National Traian din Drobeta Turnu-Severin si Filiala Mehedinti a SSMR. La concurs au participat peste 500 de elevi din Mehedinti ,Caras-Severin, Gorj, Dolj, iar la Simpozionul desfasurat cu aceasta ocazie au participat cu lucrari de istoria matematicii si articole metodico-stiintifice peste 60 de cadre didactice din Mehedinti, Gorj, Bucuresti, Timis, Dolj, Caras-Severin. Ne- a bucurat participarea din partea Biroului Consiliului National al SSMR a domnului Mircea Trifu, care in cadrul Simpozionului a sustinut o interesanta prezentare despre Matematica in limba romana din Ardeal.

SUBIECTE Clasa a IV-a

1.a) Se consideră numerele : ( ){ }1 2 1 2 1 2 1 2a ⎡ ⎤= + ⋅ + ⋅ + ⋅ +⎣ ⎦ şi ( ){ }8 7 6 5 4 3 2 1 :9b ⎡ ⎤= + + ⋅ + ⋅ + −⎣ ⎦ .

Să se calculeze ( ) ( ):a b a b+ − . b) Suma a patru numere naturale impare consecutive este egală cu 2008. Să se afle numerele.

Ştefan Marica 2. Suma a trei numere naturale este egală cu 179. Al doilea număr este cu 5 mai mare decât dublul primului număr şi cu 4 mai mic decât cel de-al treilea număr. Să se afle cele trei numere.

prelucrare G.M. 9/2007 3. Se consideră tabloul cu 100 de linii:

1 2 1 3 2 1 4 3 2 1 5 4 3 2 1 6 5 4 3 2 1 ……………………….. 100 99 98 97 ……..3 2 1

a) să se afle suma numerelor de pe linia a 10-a ; b) de câte ori apare în tablou numărul 13 ? c) să se determine numărul de apariţii al cifrei 7 în scrierea tuturor numerelor din tabloul considerat.

Manuela Prajea Clasa a V -a

1.a) Aflaţi restul împărţirii numărului A= 1∙2∙3∙4∙ ........ ∙ 2007+2008 la 2002 b) Fie A={x | x=n2+2007n+2009,n∈N} şi B={x|x=20072008n-1, n∈N}.Să se determine A∩B.

Raducan Emilia 2.Să se determine numerele naturale nenule m şi n astfel încât numărul m!+22n+1 să fie pătrat perfect,unde m!=1∙2∙3∙.....∙m.

GM Nr.11/2007 3)Un elev are la inceputul anului şcolar 7 pixuri şi 20 de creioane.Fiind foarte neglijent el pierde in fiecare zi câte două instrumente de scris din cele de mai sus.Dacă pierde două instrumente de acelaşi fel ,părinţii îi mai cumpără , în aceeaşi zi, un creion ,iar dacă pierde un pix şi un creion îi mai cumpără un pix.Care este ultimul instrument de scris ce îi rămâne elevului?

Lupu Adrian

Page 5: RMM 8.pdf

Petre Sergescu

- 4 -

Clasa a VI-a

1) Să se determine numerele naturale de forma abc care satisfac relaţia: ( ) 2008 3333 =++ cbaa .

Paponiu Dana

2) a)Să se arate că fracţia 3243

++

nn este ireductibilă pentru orice N∈n .

b)Să se determine numerele naturale m si n pentru care are loc relaţia:

N∈++

+++

121

3243

mm

nn .

Marica Stefan 3) Fie segmentul [ ]AB , BA ≠ şi 2007 puncte distincte, situate în interiorul său, care îl împart în 2008 segmente. Punctului A i se asociază numărul 0 , iar punctului B i se asociază numărul 1. Celor 2007 puncte li se asociază arbitrar unul din numerele 0 sau 1. Să se arate că, dintre cele 2008 segmente, un număr impar au extremităţile asociate cu acelaşi număr.

Prajea Manuela Clasa a VII-a

1.a)Demonstrati ca 1)9)(8)(7)(6( +++++ nnnn ∈Q, oricare ar fi n numar natural;

b)Exista numere naturale n pentru care 23 +n ∈Q ? Rodica Antonie

2.a)Fie ABCD un trapez cu bazele AB si CD, AB=b, CD=a, 0<a<b, ACIBD={O}. Dreapta d trece prin punctul O si intersecteaza bazele [ ]CD si [ ]AB in punctele E, respectiv F.Notam CE=x.Sa se determine valoarea lui x pentru care aria(ADEF)=aria(BCEF) ; b)Paralelogramul MNPQ este decupat din interiorul paralelogramului ABCD.Sa se determine o dreapta care imparte figura ramasa in doua figuri cu aceeasi arie. Justificati raspunsul.

Gheorghe Cainiceanu 3.Pentru n natural, se considera numarul real:

an= ⋅++ nn 1 ( 11 ++n 11 −− n ) a) calculati a2; b) calculati partea intreaga a numarului an , n∈N;

G.M. nr.4/2007 Clasa a VIII-a

1. a)Se consideră mulţimea: { }1001,, ≤<≤∈+= bababaA N . Câte elemente are QIA ? b)Fie [ ]AB şi [ ]CD două segmente necoplanare, M mijlocul lui [ ]AB şi N mijlocul lui [ ]CD . Să se arate că dacă CDAC ⊥ şi CDBD ⊥ , atunci CDMN ⊥ .

Gimoiu Iuliana 2. Se consideră numerele reale nenule x şi y , pentru care numerele

xyyx , şi ( ) ( )2222 11 ++++ xxxxy

sunt toate numere raţionale. Arătaţi că x şi y sunt de asemenea numere raţionale. Nedeianu Dan

3. Se consideră un cub cu muchia cm1 şi un plan care secţionează cubul după un hexagon. Să se determine valoarea minimă a perimetrului hexagonului.

Prajea Manuela

Page 6: RMM 8.pdf

Petre Sergescu

H

- 5 -

SSM Clasa a IX-a

1. Se considera cercul C(O;R) si punctele distincte A,B,C,D∈ C(O;R).Daca 0=+++ ODOCOBOA sa se arate ca A,B,C,D sunt varfurile unui dreptunghi.

Manuela Prajea 2. a). Intr-o zi un pescar a pescuit 200 de platici si 18 salai. A doua zi a prins cu 3 platici mai putin si cu 2 salai mai mult si tot asa, in zilele care au urmat cate 3 platici mai putin si cate 2 salai mai mult pana in ziua in care a prins mai multi salai decat platici. Cate platici a prins in total? b) Sa se demonstreze ca intre lungimile laturilor unui triunghi oarecare exista inegalitatea:

1<−+−+−ca

ac

bc

cb

ab

ba

.

Este adevarata inegalitatea daca a,b,c sunt numere strict pozitive oarecare? GheorgheCainiceanu ,Eleodor Popescu

3. Se considera functia f: R→ R; |)12|,3|,min(|)( −= xxxf

a). Sa se reprezinte grafic functia f. b). Sa se rezolve ecuatia f(x)=2. c). Sa se discute, in functie de parametrul real m, numarul solutiilor reale al ecuatiei f(x)=m. d). Daca }{ 4,3]2,1[ ∪=A si { } ]4,3[2,1 ∪=B sa se reprezinte intr-un reper cartezian produsele carteziene A X B si B X A.

.Daniel Sitaru ,Dan Nanuti Clasa a X-a

1. Să se rezolve ecuaţia: 26381067 3 23 23 +−−=+−+− xxxxx .

G.M. nr. 11/2007 2. a) Dacă yx <<0 şi 1>a , să se arate că:

yax yx <⋅ − b) Să se rezolve ecuaţia: tgxx =2cos2 .

Dan Nanuti 3. Fie, RCaCba −∈∈ ,, , aIm ≠ bIm . Dacă numerele complexe ababa +− ,, au acelaşi

modul, să se arate că b=0. Nedeianu Dan

Clasa a XI-a

1). a). Calculati unde , a R∈ si n ∈ .

b). Fie A ∈ (C), A≠ a. i. cu p≥5, p prim. Calculati . Eleodor Popescu ,Constantin Giugiuc

2). Consideram Nn∈ , n≥2. a). Aratati ca ecuatia are solutie unica in intervalul (0,1). b).Pentru n≥2, notam cu solutia din intervalul (0,1) a ecuatiei . Calculati nn

x∞→

lim . Cristinel Mortici

3). Definim functiile astfel: .

a).Demonstrati relatiile:

Page 7: RMM 8.pdf

Petre Sergescu

- 6 -

b). Aratati ca functia f este inversabila si , apoi calculati xxsh

x

)(lim0→

.

c). Fie sirul definit astfel: Calculati )2(4lim −

∞→ nn

na .

Leonard Giugiuc Clasa a XII-a

1. Să se calculeze limita: dxxl ∫→=

ε

ε ε 0

200720080

sin1lim

Eleodor Popescu 2. Fie f: [0,1] → [ 0, ∞ ) o funcţie continuă.

Demonstraţi că dacă şirul ∑=

⎟⎠⎞

⎜⎝⎛

+=

n

kn n

kfk

kn

a1 1

1 este convergent la zero, atunci f(x) = 0, oricare

ar fi x ∈ [0, 1]. Cristinel Mortici

3. Se consideră mulţimea ⎪⎭

⎪⎬⎫

⎪⎩

⎪⎨⎧

∈⎟⎟

⎜⎜

⎛= 7

^^

^^

^^

,| ZbaabbaM şi matricea MA ∈

⎟⎟

⎜⎜

⎛= ^^

^^

4334 .

Să se rezolve în M ecuaţia X6 = A. Manuela Prajea

Premiul I XII Gosea Victor CNCarol,Craiova, Stoian Bogdan CN G.Cosbuc,Motru, Tigora Andrei CN Traian,Drobeta XI Dragoi Maris CN G.Cosbuc,Motru, X Pasureanu Victor CN Carol.Craiova, IX Tesila Bianca CN Traian,Drobeta, VIII Nistor Adriana Gen.14.Drobeta, VII Stefan Andrei CN Traian,Drobeta, VI Puican Tiberiu CN Traian,Drobeta, Trofin Raluca CN Carol Craiova, V Pogacean Victor CN Traian,Drobeta, IV Burtea Catalin Gen.2,Drobeta, Popescu Cristina Gen.16,Drobeta Premiul II XII Mituca Atena CN Traian,Drobeta, XI Butaru Nicu CN Traian Drobeta, Posa Bogdan, CN G.Cosbuc,Motru, X Pirvulescu Dan CN Traian,Drobeta, IX Mirosu Raluca CNTraian,Drobeta, VIII Baleanu Andrei CNG.Cosbuc,Motru, VII Tanasie Denisa CNTraian,Drobeta, VI Troncota Diana CNTraian, V Calota Dragos CN Carol,Craiova, IV Paleacu Cosmin Gen.2,Drobeta, Sontea Claudiu Gen.14 Premiul III XII Coanda Oana CNTraian,Drobeta, Prundeanu Andreea CNTraian,Drobeta, XI Ciorobea Mihai ,Mema Alexandra,Nistor Ovidiu CNTraian,Drobetam, X Birovescu Georgiana CNTraian Drobeta ,Alexandru Bogdan CN Carol Craiova, IX Agape Mihai CNTraian,Drobeta, VIII Andreescu Madalina CNTraian,Drobeta, Semenescu Anca CDLoga,Caransebes, VII Anghel Cristian V Gomoiu ,Vinju-Mare, Gimoiu Ruxdandra CNTraian Drobeta, VI Benga Andrei CNTraian Drobeta, V Chirita Dan CNTraian,Drobeta, IV Butaru Madalina Gen.14,Drobeta,Craciun Madalina Gen.2, Drobeta, Dragotescu Radu Gen.6,Drobeta, Filip Radu Gen.6,Drobeta, Lica Robert Gen.2,Drobeta, Tanasie Danut Gen.2,Drobeta, Balu Smaranda Gen.2, Drobeta

Page 8: RMM 8.pdf

Petre Sergescu

H

- 7 -

SSM SIMPOZIONUL NATIONAL PETRE SERGESCU 21.03.2008

EDITIA A IV-A MIRCEA TRIFU Secretar general al SSMR,Bucuresti

Aritmetica in limba romana din Ardeal CHIRCU MARIANA DOINITA Inspector general ISJ

Matematici elementare : aspecte metodice prin aplicatii practice CAINICEANU GHEORGHE, CNT,

Asupra numerelor algebrice PRAJEA MANUELA CNT,

Un criteriu de tip L’Hospital pentru calculul unor limite de functii STOICA RODICA CNT,

Stiinta si constiinta-moralitatea gandirii creatoare GHIMES DANIELA Inspector ISJ Mehedinti

Din viata si activitatea matematicianului roman Traian Lalescu ANTONIE RODICA MIHAELA CNT,

Triunghiul ortic PAPONIU DANA ,CNT,

Polinoame ireductibile MARICA STEFAN Prof.Pensionar CNT, Colegiul National Traian

In amintirea unor fosti elevi DAN NEDEIANU Colegiul Tehnic D-l Tudor

Aplicatii ale teoremei Lagrange in rezolvarea unor ecuatii exponentiale GIMOIU IULIANA CNT

Proprietatea lui Darboux VIOREL SAHAGIA Profesor pensionar CNT

Aspecte privind generalul si particularul in matematica LUPU ADRIAN Col.Tehnic Decebal

Congruente modulo n in gimnaziu si liceu GHIOCEL FLORIN Inspector ISJ

Matematicieni europeni din mileniul III GHIOCEL GEORGETA GSI-AUTO

Mobilitate europeana prin programe comunitare GIUGIUC LEONARD CNT

Aplicatii ale criteriului Cesaro-Stoltz GIUGIUC CONSTANTIN Col.Tehnic D-l Tudor

Metode de rezolvare pentru determinanti ANTONIE NICUSOR Lic.Stefan Odobleja

Aplicatii ale calculului integral in fizica IONICA CONSTANTIN Sc.Gen.14

Metoda reducerii la absurd BEJENARU LAVINIU CNT

Siruri cu aspecte speciale BALOI VICTORIA,BALU NICOLETA,Lic.Pedagogic St.Odobleja,

Vocatia formatoare a scolii MIHAELA POPESCU Institutor Sc.Gen.2, POPESCU DUMITRU Inst.Sc.Gen.24 Craiova

Noi modalitati de abordare a lectiei de matematica PIT-RADA MARICA Sc.gen.Nr.5

O metoda de calculare a distantei dintre doua drepte neparalele in spatiu PIT-RADA VASILE CNT

Despre spatii proiective si teoria codurilor PIPINA CAMENITA Institutor Sc.Gen.3

Gheorghe Titeica-matematician de seama al judetului Mehedinti ALINA TEIS Inst.Liceul Pedagogic Stefan Odobleja

Jocul didactic, modalitate de optimizare a procesului de predareinvatare la ciclul primar GORUN SANDA Lic.Traian Lalescu, Orsova

Probleme cu continut practic de aflare a extremelor OPRITA MANUELA Col.Tehn.Decebal

Teoreme din geometrie demonstrate trigonometric MIHAIL MARIA,MIHAIL ANDREI, Institutori Sc.Gen.6

Stimularea creativitatii prin orele de matematica

Page 9: RMM 8.pdf

Petre Sergescu

- 8 -

COCOARA VICA Obarsia Closani Aspecte metodice in rezolvarea unor probleme de aritmetica

OSIAC VIOREL Inginer SC Termo. SA Dan Barbilian- intre matematica si poezie

PLOTOGEA JANETA ,Col.Tehnic Decebal Polinoame ireductibile cu coeficienti intr-un corp

BONDOC GABRIELA Grup Scolar Auto Cateva considerente despre functii monotone si continue

LUGOJ EUGEN CNT Interdisciplinaritatea matematica-chimie in rezolvareaproblemelor de chimie

NANUTI DAN CNT, SITARU DAN Col.Economic Caracterizarea structurilor de Grupuri finite utilizand numarul de automorfisme de corp

ROATES MISU CNT,ROATES OLGA CNT Matematica si adolescenta

NISTOR CORNELIA CNT Matematica aplicata in biologie

RADUCAN EMILIA STEFANIA Col.Tehn.Decebal Structuri algebrice. Grupuri.

BIZDOACA CLAUDIA Col.Economic Utilizarea polinoamelor ciclotomice in rezolvarea ecuatiilor algebrice cu coeficienti complecsi

HRUBY ELISABETA CNT Statistica matematica si literatura

TOMA NINETA CNT, Poezia –geometrie a formelor

OSAIN VICTORIA Strehaia Patrate si cuburi de numere intregi

TRUSCA DANIELA CNT Paradoxuri si probabilitati in poezie

BADESCU OCTAV CNT Mituri si cauze in schimbarile climatice

LUGOJ TANTA Col.Tehnic Decebal Aspecte metodice privind Descoperirea, motivarea si interpretarea inegalitatii lui Cauchy si a coeficientului de corelatie cu aplicatii in diverse domenii

ZAGARA GABRIELA CNT Limbaje formale

ZAICU RODICA, DRAGHIA MADALINA Institutori CN G.Cosbuc Motru Modalitati de optimizare a tehnologiei didactice la matematica

LICA MARIELA CAMELIA CNT Matematica-Muzica a ratiunii

CONSTANTINESCU CERASELA CNT Mari matematicieni romani

CIVITU LUISA CNT Petre Sergescu-personalitate multilaterala

FLEANCU RALUCA CNT Petre Sergescu-ambasador de renume al Romaniei in lume

TRAILESCU GOGAN DIANA VERONICA ,Obarsia Closani Teorema lui Sylvester si aplicatii

COVASALA SORIN Obarsia Closani Cercul lui Euler

POPESCU OCTAVIAN TUDOR Colegiul F.Nitti.Timisoara Previziune economica prin metoda autoregresiva

SAMFIRESCU MIRCEA CNT Sisteme dinamice deterministe.Haos determinist.

OPROIU MIHAELA CNT Pitagora si pitagorismul

CATANA ELENA CNT Aristotel-omul si opera

HUMITA DORINA,MIRULESCU MARITA, CD Loga,Caransebes Grigore Moisil-O legenda a matematicii romanesti

MANDRESI ANA CD Loga Caransebes Formarea si dezvoltarea competentelor prin studiul teoremelor de medie

RAESCU GHEORGHE,Orsova Probleme deosebite de geometrie

Page 10: RMM 8.pdf

Petre Sergescu

H

- 9 -

SSM GHINEA MARIA CNT

Numere simbolice in crestinism FARAGO AFRODITA Orsova

Matematicieni romani-Dimitrie Pompeiu FARAGO ALEXANDRU Orsova

Traian Lalescu DRULA ILEANA Strehaia

Inegalitati algebrice DRULA MIHAI Strehaia

Bazele psihopedagogice si metodologice ale rezolvarii problemelor MALINEANU GABRIELA Sc.Gen.4

Legarea teoriei de practica CHIRFOT CARMEN VICTORITA Col.Tehnic D-l Tudor

Aplicatii care folosesc relatia lui Legendre pentru calcularea exponentului numarului prim CRISAN LIVIA Col.Tehn.Lorin Salagean

Aspecte metodice privind predarea-invatarea proiectiilor ortogonale pe un plan NITOIU ANGELA Col.Tehnic Decebal

Functia homografica ROMAN CAMELIA CNT

Inteligenta artificiala STIOIU FLOAREA,GRIGORAS ANISOARA Sc.Gen.14

Matematicieni celebri MOCLEA ADRIANA Sc.Gen.11

Importanta constructiilor geometrice in rezolvarea problemelor de geometrie ROMAN ANA Institutor Sc.Gen.11

Importanta rezolvarii problemelor de geometrie in ciclul primar BASULESCU MARIA Institutor Sc.Gen.11

Formarea deprinderilor de rezolvare a problemelor la ciclul primar ICA FLORENTINA Institutor I.St.Paulian

Introducerea notiunilor matematice si dezvoltarea abilitatilor logice prin utilizarea jocului didactic la ciclul primar

MEDRAGONIU CRISTIANA Institutor,Gen.11 Rolul jocului didactic matematic in lectiile de matematica

BADOIU EMILIA CNT Matematici financiare in invatamant

PAUNESCU LIVIA CNT Elemente de statistica in organizarea bazelor de date

DRAGOMIR VETURIA CNT Programe soft pentru calcul financiar

CIOBOTA NORICA CNT Experimente practice de fizica matematica

Page 11: RMM 8.pdf

NOTE MATEMATICE

- 10 -

Pregatire pentru olimpiadele, concursurile şi barajele de matematică

prof.dr.Manuela Prajea, Colegiul Naţional „Traian”

Prezenta rubrică vine în sprijinul elevilor care se pregătesc pentru olimpiade, concursuri şi baraje la matematică, prin expunerea unor probleme inedite prin originalitatea raţionamentului, date la concursurile naţionale sau regionale pe mapamond. Problemele expuse sunt însoţite de soluţii comentate, menite să permită elevului să surprindă atât mecanismul unui raţionament adecvat cât mai ales posibilitatea sugerării treptate a paşilor logici, elevul fiind astfel invitat să găsească prin forţe proprii continuarea ideilor de rezolvare. În acest sens am marcat corespunzător ideile care invită la „spargerea problemei”. Se consideră mulţimea { }2 1,2,3,..., 2 1,2n n n= −� şi submulţimile sale disjuncte

{ }1 2 3,, , ..., ,na a a a { }1 2 3,, , ..., nb b b b astfel încât 1 2 3 ... na a a a> > > > şi 1 2 3 ... nb b b b< < < < .

Să se arate că are loc egalitatea 21 1 2 2 ... n na b a b a b n− + − + + − = .(identitatea lui

Prozvolov) Soluţie: Partiţionarea mulţimii 2n� conform enunţului sugerează raportarea termenilor secvenţelor ( ) 1,i i na

= şi ( ) 1,i i nb

= la partiţia canonică { }1,2,...,X n= şi { }1, 2,...,2Y n n n= + + .

Dificultatea constă în a observa că dacă ka X∈ atunci kb Y∈ şi dacă ka Y∈ atunci kb X∈ ,

1,k n= . Într-adevăr, dacă prin absurd, există ,k ka b X∈ atunci ,k ka n b n≤ ≤ . În această situaţie, un număr de cel puţin 1n k− + termeni ai secvenţei ( ) 1,i i na

= vor aparţine

mulţimii X şi analog un număr de k termeni ai secvenţei ( ) 1,i i nb = vor aparţine din nou

aceleaşi mulţimi X . Astfel mulţimea X va conţine cel puţin 1n + numere naturale distincte -contradicţie. Analog, cazul ,k ka b Y∈ conduce la contradicţie. Suntem în măsură să calculăm suma cerută. Având în vedere cele de mai sus conchidem că pentru orice pereche ( ),k ka b , unul dintre numere aparţine mulţimii X şi celălalt mulţimii Y . Deci:

( ) ( ) [ ]1 1 2 2 ... 1 2 ... 2 1 2 ...n na b a b a b n n n n⎡ ⎤− + − + + − = + + + + + − + + +⎣ ⎦2...n n n n= + + + = .

În spiritul identitaţii Prozvolov propunem spre rezolvare următoarea : Se consideră { }1 2 3,, , ..., ,na a a a { }1 2 3,, , ..., nb b b b două submulţimi disjuncte ale mulţimii

{ }1,2,...,2n , *n∈� astfel ca 1 2 3 ... na a a a> > > > şi 1 2 3 ... nb b b b< < < < .

Pentru orice 1,k n= definim max ,k kk

k k

a bb a

α⎧ ⎫

= ⎨ ⎬⎩ ⎭

.

Să se arate că 1 2... nα α α ∈� .

Page 12: RMM 8.pdf

NOTE MATEMATICE

H

- 11 -

SSM

Probleme pentru Olimpiada elev. Ioana Ionas

prof. dr. Daniel Stretcu

Probleme propuse pentru clasa a VI-a

1. Arătaţi că oricare trei numere prime, mai mari sau egale cu 13, se pot alege două astfel încât suma sau diferenţa lor să se dividă cu 12.

Soluţie: Din teorema împărţirii cu rest, un număr prim, mai mare sau egal cu 13 este de forma

12k+1, 12k+5, 12k+7 sau 12k+11, cu alte cuvinte de forma 12k± 1 sau 12k± 5. Folosind principiul cutiei, fiind date 3 numere prime, vor exista 2 numere din aceeaşi clasă a căror diferenţă sau sumă este evident divizibilă cu 12.

2. Câte cifre are numărul cifrelor numărului 20082008 ? Soluţie: Notăm cu N(a) numărul cifrelor numărului a. Avem: 1000<2008<10000 ⇒ 10 20084200820083 102008 ⋅⋅ << ⇒ ⇒ )10()2008()10( 20084200820083 ⋅⋅ ≤≤ NNN ⇒

⇒ 8032)2008N(6024 2008 ≤≤ ⇒ )8032(N))2008(N(N)6024(N 2008 ≤≤ ⇒

⇒ 4))2008(N(N4 2008 ≤≤

Deci 4))2008(N(N 2008 = .

3. Pe o tablă 33× avem un singur pătrăţel negru într-un

colţ, iar celelalte sunt albe. Vom numi recolorare operaţia de schimbare a culorilor în toate pătratele unei linii sau coloane.

Demonstraţi că tabla nu poate fi transformată într-o tablă complet albă recolorând, una câte una, câteva linii şi câteva coloane.

Soluţie:

Demonstrăm prin reducere la absurd. Presupunem că tabla ar putea deveni toată albă după un anumit număr de recolorări. Punem în pătrăţelele negre -1, iar în pătrăţelele albe numărul 1. Luăm ca invariant produsul numerelor din pătrăţelele haşurate în figura alăturată.

Iniţial, produsul este -1 şi după fiecare recolorare rămâne neschimbat. Deci, oricâte recolorări am face nu poate fi egal cu 1, ca în cazul când toate pătrăţelele ar fi albe şi am obţinut o contradicţie.

Page 13: RMM 8.pdf

NOTE MATEMATICE

- 12 -

Probleme pentru Olimpiada prof. dr. Daniel Stretcu

Probleme propuse pentru clasa a XI-a 1. Fie A∈M2 (R) o matrice pătratică ce are proprietăţile 1)det( 2 =− IA şi

3)det( 2 =+ IA . Să se calculeze:

a) )4det( 2IA ⋅− şi )4det( 2IA ⋅+ b) )65det( 24 AAA ⋅+⋅−

Soluţie: a) Fie baXXAIXXP +⋅−=−⋅= 2

2 )det()( , unde )(XP este polinomul caracteristic al matricei A cu Aa det= şi )(Atrb = .

Folosind )det()det()1()det( 222

2 IXAIXAAIX ⋅−=⋅−⋅−=−⋅ , avem baIAP +−=−= 1)det()1( 2 . Dar 1)det( 2 =− IA , din ipoteză, rezultă ba = )1( . Analog

baIAP ++=+=− 1)det()1( 2 , iar 3)det( 2 =+ IA . Deci 2=+ ba )2( . Din )1( şi )2( ⇒ 1)(1 2 +−=⇒== XXXPba .

13)4()4det( 2 ==⋅− PIA . 21)4()4det( 2 =−=⋅+ PIA .

b) Avem 222 OIAA =+− (din Teorema lui Hamilton-Cayley).

2242

24

22 2)( IAAAIAAIAA +⋅−=⇒−=⇒−= )3(

Folosind în )65det( 24 AAA ⋅+⋅− relaţia )3( , rezultă =+⋅+⋅−=⋅+⋅−+⋅−=⋅+⋅− )44det(]65)2det[()65det( 2

222

224 IAAAAIAAAAA25)5det(]4)(4det[ 222 =⋅=+⋅+−−= IIAIA .

2. Fie şirul 0)( ≥nnx , definit prin relaţia 2516

1 ++

=+n

nn x

xx , ∈ n)( ℕ şi 10 >x .

Să se arate că şirul 0)( ≥nnx este convergent şi să se calculeze nnx

∞→lim .

Soluţie: Metoda I:

Fie ⎟⎟⎠

⎞⎜⎜⎝

⎛=

2516

A matricea asociată şirului omografic 0)( ≥nnx .

Din teorie cunoaştem că dacă λ1, λ2 sunt rădăcinile ecuaţiei:

0det)(2 =+⋅− AAtr λλ )1( cu 21 λλ ≠ atunci 221

12

11

21

21 det IAAAnnnn

n ⋅−−

⋅−⋅−−

=−−

λλλλ

λλλλ .

⎟⎟⎠

⎞⎜⎜⎝

⎛=

2516

A ⇒ 8)( =Atr şi 7det =A .

Relaţia )1( devine 0782 =+− λλ ⇒ 11 =λ şi 72 =λ .

Page 14: RMM 8.pdf

NOTE MATEMATICE

H

- 13 -

SSM

2

1

71717

7171 IAA

nnn ⋅

−−

⋅−⋅−−

=−

⇒ 2

1

6177

617 IAA

nnn ⋅

−⋅−⋅

−=

⎟⎟⎠

⎞⎜⎜⎝

+−−+⋅

=57)17(517175

61

nn

nnnA ⇒

57)17(517)175(

0

0

++−⋅−++⋅

= nn

nn

n xxx

11515

]751)

711(5[7

]711)

715[(7

lim57)17(517)175(

limlim0

0

0

0

0

0 =++

=++−

−++=

++−⋅−++⋅

=∞→∞→∞→ x

x

x

x

xxx

nnn

nnn

nnn

nn

nnn.

Metoda a II -a Determinăm an, bn, cn, dn astfel încât:

nn

nnn dxc

bxax+⋅+⋅

=0

0 cu

⎩⎨⎧

========

2,5,1,61,0,0,1

1111

0000

dcbadcba

2516

1 ++

=+n

nn x

xx ⇒ 25

16

0

0

0

0

1

++⋅+⋅

++⋅+⋅

⋅=+

nn

nn

nn

nn

n

dxcbxadxcbxa

x ⇒nnnn

nnnnn dxcbxa

dxcbxax2255

66

00

001 +⋅++⋅

+⋅++⋅=+ ⇒

⎪⎪⎩

⎪⎪⎨

+=+=+=+=

⎪⎪⎭

⎪⎪⎬

+⋅+⋅

=

++++++

=

+

+

+

+

++

+++

+

nnn

nnn

nnn

nnn

nn

nnn

nnnn

nnnnn

dbdcac

dbbcaa

dxcbxax

dbcaxdbcaxx

2525

66

25)25(6)6(

1

1

1

1

101

1011

0

01

Rezolvând acest sistem

⎪⎪⎪⎪

⎪⎪⎪⎪

+=

−=

−=

+⋅=

)57(61

)17(65

)17(61

)175(61

nn

nn

nn

nn

d

c

b

a

57)17(517)175(

0

0

++−⋅−++⋅

= nn

nn

n xxx

11515

]751)

711(5[7

]711)

715[(7

lim57)17(517)175(

limlim0

0

0

0

0

0 =++

=++−

−++=

++−⋅−++⋅

=∞→∞→∞→ x

x

x

x

xxx

nnn

nnn

nnn

nn

nnn

Page 15: RMM 8.pdf

NOTE MATEMATICE

- 14 -

ASUPRA NUMERELOR ALGEBRICE

Prof.dr. Gheorghe Cainiceanu,CNT

INTRODUCERE Analizand numerele reale sau complexe dupa criteriul de a fi sau a nu fi radacinile unor polinoame, C.H.Hermite (1822-1901) si F.Lindemann (1852-1939) au introdus notiunile de numere algebrice si numere transcendente. Nota de fata isi propune prezentarea acestor numere si a unor exemple accesibile la nivel elementar.

NOTIUNILE DE NUMAR ALGEBRIC SI NUMAR TRANSCENDENT DEFINITIA 1 Daca un numar a ∈C este radacina a unei ecuatii de tipul xn+a1xn-1+…+an =0 cu coeficienti rationali, a se numeste numar algebric. EXEMPLE 1) orice numar rational a este algebric deoarece este solutia ecuatiei x-a=0. 2) 2 este algebric deoarece este solutie a ecuatiei x2-2=0 ,ecuatie cu coeficienti rationali. In 1873 C.H.Hermite demonstreaza primul ca e (baza logaritmului natural) nu este numar algebric. Ne propunem aici sa aratam doar ca e nu este numar rational. PROPOZITIE numaeul lui Euler ,e, nu este rational. Demonstratie. Pentru sirul

!

1.....!2

1!1

11n

sn ++++=

Se arata prin calcule ca 0<e-sn =

.1,!1....]

)1(1

111[

)!1(1....

)!2(1

)!1(1

2 ≥=++

++

++

<++

++

nnnnnnnn

Deci

)1,0(,!!

1.....!2

1!1

11 ∈+++++= nn

nnne φ

φ.

Presupunem acum ca qpe = .Deducem ca exista )1,0()( ∈qφ ,pentru care avem

qqq

qqp

!)(

!1...

!21

!111 φ

++++= .

Prin inmultirea convenabila, gasim

Zqqq

pqq ∈++−= !)!

1...!1

11(!)(φ , ceea ce este absurd.

In 1882 F.Lindemann a demonstrate ca π nu este algebric. DEFINITIE. Daca numarul complex a nu este algebric el se numeste transcendent. OBSERVATIE O consecinta a faptului ca π este transcendent, este aceea ca nu putem construe cu rigla si compasul un segment egal cu lungimea cercului, sau un patrat echivalent cu un cerc. PROPOZITIE Numerele algebrice formeaza un corp in raport cu operatiile de adunare si inmultire induse di multimea numerelor complexe. DEFINITIE Fie K⊂ L corpuri commutative astfel incat K este subcorp al lui L. spunem ca L este o extindere a lui K. EXEMPLU C este o extindere a lui R.

Page 16: RMM 8.pdf

NOTE MATEMATICE

H

- 15 -

SSM

PROPOZITIE Fie K un corp comutativ si f ∈K[X] ,grad f≥1. Exista o extindere a lui K , notata L, astfel incat f sa aiba cel putin o radacina in L. Demonstratie [1]. COROLAR Fie K un corp comutativ si f ∈K[X] ,grad f ≥ 1.Atunci exista o extindere a lui K in care f sa aiba toate radacinile. DEFINITIE se numesc corpuri de numere algebrice extinderile finite ale corpului numerelor rationale, iar elementele lor se numesc algebrice. DEFINITIE Un numar algebric se zice intreg daca satisface o ecuatie algebrica monica cu coeficienti intregi. EXEMPLE

1) Numarul 8πctg este intreg algebric.

Justificare.Prin calcul elementar si cunoscut, deducem ca 8πctg = 12 + .O ecuatie cu

coeficienti intregi care admite acest numar ca solutie este ecuatia x2-2x-1=0.

2) Numarul

8sin

este intreg algebric.

Justificare. Se arata ca acest numar este solutie a ecuatiei x4-8x2+8=0, deci este numar intreg algebric. BIBLIOGRAFIE [1] C.Nastasescu,C.Nita,C.Vraciu Bazele Algebrei Editura Academiei 1986 [2] J.Kurschak Probleme de matematica pentru concursurile din Ungaria, EDP, 1972 [3] Z.I.Borevici, I.R.Safarevici, Teoria Numerelor, Ed.St.Enciclopedica,Bucuresti 1985. [4] Gh.Siretchi Calcul diferential si integral , Ed.St.Enciclopedica, Bucuresti 1985.

Page 17: RMM 8.pdf

CERCUL DE MATEMATICA

16- 16 -

Tema pentru grupa de performanta la clasa a- V -a

Operaţii cu numere naturale binare Prof. Chirfot Carmen – Victoriţa

Prof. Chilea Ion Colegiul Tehnic „Domnul Tudor”

În această notă, vom enumera şi exemplifica câteva dintre operaţiile de bază care se pot aplica numerelor scrise în baza 2. Aşa cum ştim, cifrele bazei de numeraţie 2 sunt 0 şi 1. Operaţiile de adunare, scădere, înmulţire şi împărţire sunt similare celor din baza de numeraţie 10, cu specificaţia că se va ţine cont de câteva reguli enumerate în acest articol.

1. Transformarea unui număr natural din baza 10 în baza 2 Se împarte numărul natural n la 2, apoi câtul obţinut se împarte din nou la 2 şi algoritmul

continuă până când obţinem câtul 1. Se reţine ultimul cât şi resturile obţinute la toate aceste împărţiri în ordine inversă, forma obţinută reprezentând scrierea în baza 2 a numărului n. 2. Transformarea unui număr natural din baza 2 în baza 10

Să presupunem că numărul natural scris în baza 2 este *

1)2(321 ,0,,1},1,0{,... Npapiaaaaan ip ∈≠=∈= . Atunci, în baza 10 avem:

*11

22

11 ,0,,1},1,0{,2...22 Npapiaaaaan ipp

pp ∈≠=∈+⋅++⋅+⋅= −−− .

3. Reguli de adunare în baza 2

21011;101;110;000 =+=+=+=+ .

4. Reguli de scădere în baza 2

1110;011;101;000 )2( =−=−=−=− .

5. Reguli de înmulţire în baza 2

111;101;110;000 =⋅=⋅=⋅=⋅ .

6. Reguli de împărţire în baza 2

11:1;11:0 == . Împărţirea la 0 nu poate fi efectuată,

aşa cum nici în baza 10 nu există (de fapt, nu există în nici

o bază de numeraţie).

Exemple şi exerciţii

a) Transformaţi în baza 2 numărul 73.

Soluţie: Împărţim repetat numărul natural 73 la 2. Se reţine

ultimul cât şi resturile obţinute la împărţiri se citesc de la

dreapta la stânga. Prin urmare, )2()10( 100100173 = .

b) Transformaţi numărul natural )2(1001001 în baza 10.

73

72

2

36 2

=1 36 18

18

2

=0 9

8

2

= 0 4

4

2

1 2

2

2

0 1

0

Page 18: RMM 8.pdf

CERCUL DE MATEMATICA

H SSM

- 17 -

Soluţie: 73186412020212020211001001 23456)2( =++=+⋅+⋅+⋅+⋅+⋅+⋅= .

c) Calculaţi )2()2( 111111 + .

Soluţie: Vom efectua adunarea în scris, asemenea celei din baza 10, dar ţinând cont de regulile

de adunare în baza 2:

)2(

)2(

)2(

01001

111111 +

•••

Observaţie: Punctele scrise deasupra

cifrelor binare sunt acele cifre pe care le ţinem în minte, atunci când rezultatul depăşeşte cifra maximă în baza 2 şi aceste puncte sunt de valoare egală cu 1 şi se vor aduna cifrelor de pe coloana asociată acestora.

d) Calculaţi )2()2( 111001 − .

Soluţie:

)2(

)2(

)2(

011

111001

=

−→→

Observaţie: Săgeţile de deasupra cifrelor binare

desemnează ca s-a făcut un împrumut de o unitate la cifra care are săgeată deasupra, deci din

cifra de sub săgeată se scade 1 în baza 2.

e) Calculaţi )2()2( 111111 ⋅ .

Soluţie:

)2(

)2(

)2(

)2(

)2(

101101

11111111

111111

+

f) Calculaţi )2()2( 11:1001 .

Soluţie:

1001(2)

11

11(2)

11(2)

= =11

11

= =

Exerciţii propuse pentru rezolvare

1) Efectuaţi următoarele operaţii )2()2(

)2()2(

111111001111111001

+;

)2()2(

)2()2(

111:101010

111111001 ⋅.

2) Rezolvaţi ecuaţia

)2()2()2()2()2( 1000100010011]1001101[ =+⋅+⋅x , fără a

face transformări în baza 10.

3) Transformaţi în baza 2 numărul 256.

4) Transformaţi în baza 2 numărul Nnn ∈,2 .

5) Transformaţi în baza 10 numărul Nnn

∈,000...00010oride43421 .

6) Generalizând, efectuaţi )7()7( 3334 + , fără a face

transformări în baza de numeraţie 10.

Page 19: RMM 8.pdf

CERCUL DE MATEMATICA

18- 18 -

Tema pentru grupa de performanta la clasa a- V -a

Prof. Adi Lupu, Dr. Tr. Severin

1. Să se afle n Є N astfel încât 200920082 200920089920 +=++ nn

Soluţie:

.11)1(20092008 4442009

4492002008 4 +=++=++=+ MMMMM M Ecuaţia se rescrie:

,2)10(11)10(12009200810020 42

42200920082 +=+⇒++=+⇒++=++ MnMnnn care nu

are soluţie, pentru că un pătrat perfect nu poate fi decât de formele M4 sau M4+ 1.

2. Rezolvaţi in multimea numerelor naturale ecuaţia 2007)2007(2008 20082008 ++= yyx

Soluţie:

Observăm că pentru x = 0 nu se obţin soluţii. Pentru x Є N* numărul 2008x este par. Cum

y2008 şi y2008+2007 au parităţi diferite => y2008(y2008+2007) + 2007 este impar.

În final: S = Ø

3. Rezolvaţi in Z ecuatia: 20082007 ...... 3210 – 4 = n2008(n2008 + 4)

Soluţie:

Ecuaţia se rescrie: 20082007 ...... 3210 = (n2008 + 2)2

Membrul stâng se divide cu 10, dar nu cu 100, deci nu este pătrat perfect.

În final S = Ø.

4. Dacă ma, mb, mc sunt lungimile medianelor triunghiului oarecare ABC de laturi AB = c, AC

= b, BC = a, să se arate că fractia cbammm cba

++++ este subunitară.

Soluţie:

Prelungim mediana AM a triunghiului ABC (M Є (BC)) cu segmentul MA`=AM. În

paralelogramul ABA`C rezultă că AB = A`C.

În triunghiul AA`C: 2AM < AC + CA` => AM < 2

cb+ => 2

cbma+

< .

Analog 2

camb+

< , 2

bamc+

< => 1<++++

=>++<++cbammmcbammm cba

cba

Page 20: RMM 8.pdf

CERCUL DE MATEMATICA

H SSM

- 19 -

5. Într-un triunghi oarecare lungimile laturilor sunt numerele a,b,1, unde a,b Є N*. Să se arate

că a – b se divide cu 20092008.

Soluţie:

Avem 0 ≤ |a - b| < 1. Deoarece a,b Є N* ⇒ |a-b| Є N. Deci |a - b| = 0 => a – b = 0 =>

(a – b) M 20092008

G.1. Să se arate că dacă a1, a2, …, a2009 sunt numere naturale astfel încât a1+a2+…+a2009 este par atunci 2

200922

21 ... aaa +++ este par.

Soluţie: Presupunem că 22009

22

21 ... aaa +++ este impar⇒ (a1+a2+…+a2009)+

+( 22009

22

21 ... aaa +++ )=(a1+ )(...)() 2

20092009222

21 aaaaa +++++ =a1(a1+1)+a2(a2+1)+…+

+a2009(a2009+1) este impar contradicţie căci fiecare termen este par, ca produs a două numere consecutive. G2. Ştiind că penultima sa cifră este 9 să se arate că ultima cifră a unui număr natural pătrat perfect este 6. Soluţie: Ultima cifră a unui pătrat perfect poate fi: 0, 1, 4, 5, 6, 9. Ţinând cont că un pătrat perfect nu poate fi decât de una din formele 4k sau 4k+1, k∈N, obţinem că ultima cifră căutată este 6. G3. Dacă numerele reale strict pozitive x, y, z sunt dimensiunile unui paralelipiped dreptunghic şi verifică egalitatea: x2+y2+z2+x+y+z=6 să se arate că aria totală a acestuia nu poate fi mai mare strict decât 6. Soluţie: Concluzia se rescrie: 2(xy+yz+xz)≤6⇔xy+yz+xz≤3. Presupunem că xy+yz+xz>3. Din x2+y2+z2≥xy+yz+xz⇒x2+y2+z2>3 (1). Apoi (x+y+z)2=x2+y2+z2+2(xy+yz+xz)>3+2⋅3 =9⇒x+y+z>3 (căci x, y, z>0) (2). Din (1)+(2)⇒x2+y2+z2+x+y+z>6 contradicţie. G4. Dacă într-un triunghi o latură e media aritmetică a celorlalte două şi o înălţime e media geometrică a celorlalte două să se arate că triunghiul dat are toate inaltimile congruente. Soluţie: Cu notaţiile uzuale, să presupunem a≤b≤c (1). Cum 2S=aha=bhb=chc obţinem ha≥hb≥hc (2)

dar şi cSh

bSh

aSh cba

2,2,2=== .

Din ipoteză, dacă ⇒+

=2

cba 2a=b+c.

Dar a≤b, a≤c⇒2a≤b+c. Deci trebuie să avem egalitate în fiecare din inegalităţile (1) adică triunghiul

să fie echilateral. Analog pentru c=2

ba + . Rămâne cazul b= ⇒+2

ca a+c=2b (3). Analog din (2)

rămâne de studiat cazul hb= ⇒⋅ ca hh ⇒=⋅=acS

cS

aS

bS 2222 b= bbcaac ==

+≤

22

2

)4(. Pentru a

avea egalitate în inegalitatea mediilor (4) trebuie să avem a=c cba ==⇒)1(

.

Page 21: RMM 8.pdf

CERCUL DE MATEMATICA

20- 20 -

Tema pentru grupa de performanta la clasa a- VII -a

Operaţii cu numere iraţionale Profesor Nedeianu Dan

Colegiul Tehnic “Domnul Tudor”

În cele ce urmează ne propunem să stabilim diverse legături între anumite numere iraţionale

sau raţionale, folosind formulele de calcul prescurtat şi diverse procedee. De la bun început se ştie că suma, diferenţa, produsul, câtul a oricăror două numere raţionale este tot un număr raţional (în limbaj matematic, se spune că Q este parte stabilă a lui R în raport cu operaţiile respective).

În schimb suma (diferenţa, produsul, câtul) dintre două numere iraţionale poate fi atât număr raţional cât şi număr iraţional. În acest sens, avem de exemplu că:

Q∈=−+ 0)2(2 , deşi 2 , - 2 ∉Q

Q∈=−−+ 2)12()12( , deşi 2 +1, 2 -1∉Q

Q∈−=+− 1)23)(23( , deşi Q∉+− 23,23

32

18= , deşi Q∉2,18 şi apoi

32,32,32,32 ⋅−+ sunt iraţionale cu

Q∉3,2 . Cu alte cuvinte, mulţimea QR − nu este parte stabilă a lui R în raport cu operaţiile

amintite. În continuare prezentăm o listă utilă de tipuri de probleme publicate în reviste sau date la

anumite concursuri şcolare, care se încadrează în coordonatele acestor considerente. Exemplul 1

Dacă *, Ryx ∈ sunt distincte cu xy , yx , 33 yx − Q∈ să se arate că x şi y sunt numere raţionale.

(Profesor Nedeianu Dan, Concursul ”Arhimede", 2006, clasa a VIII-a)

Soluţie: Din Qxy∈ şi Qyx∈ ⇒ Qx ∈2 şi Qy ∈2 . Apoi Qyxyxyxx ∈⋅−=− 22233 )()()( şi

cum *33 Qyx ∈− , rezultă Qx∈ şi deci Qy∈ . Condiţia yx ≠ este esenţială, pentru că 2== yx verifică enunţul fără a fi numere raţionale. Exemplul 2 Determinaţi numerele iraţionale x pentru care numerele xx 22 + şi xx 63 − sunt simultan numere raţionale.

(OJM 2008, clasa a VIII-a) Soluţie: Notând xxa 22 += , xxb 63 −= , avem Qba ∈, şi axab 2)2( −−= , şi cum Qx∉ ,

deducem că 2=a şi ab 2−= , adică 0222 =−+ xx , cu 31+−=x .

Page 22: RMM 8.pdf

CERCUL DE MATEMATICA

H SSM

- 21 -

Exemplul 3 Fie 0,, * ≠+∈ baRba , astfel încât numerele )2( 22 aba − şi )2( 22 bab − , 33 ba + sunt numere

raţionale. Să se arate că ba

ba+

22

este număr raţional.

(Profesor Nedeianu Dan, RMT 4/2003) Soluţie: Se observă că )2( 22 aba − )2( 22 bab −+ = )()(2 33 babaab +−+ şi atunci Qbaab ∈+ )( (1). Însă Qbabababa ∈+=−+− 66222222 )]2([)]2([ şi deci Qbababa ∈+−+= )()(2 6623333 ,

adică Qba ∈33 (2). Din (1) şi (2) ⇒ ba

ba+

22

∈Q

Se observă că 3 16

2=a , 3 4=b verifică enunţul, fără ca a şi b să fie raţionale.

Exemplul 4 Să se arate că dacă Rx∈ pentru care 16x şi 9x sunt numere raţionale, atunci x este număr raţional.

(OLM Bucureşti 2003, clasa a VIII-a) Soluţie: Avem 151699 =⋅−⋅ şi atunci 99 )(x , Qx ∈516 )( , deci Qxxx ∈=8081 : . Exemplul 5 Să se arate că există o infinitate de numere iraţionale yx, cu proprietatea că Nxyyx ∈=+ .

(OJM 2006, clasa a VIII-a) Soluţie: Avem xyyx 4)( 2 ≥+ şi notând 5≥⇒==+ nnxyyx .

Din nnnxxnxnxny 4)2()( 22 −=−⇒−=⇒−= , adică )4(21 2 nnnx −±= ; însă pentru

5≥n , avem 222 )2(4)3( −<−<− nnnn şi atunci Qx∉ .

Astfel putem alege ).4(21),4(

21 22 nnnynnnx −−=−+=

Exemplul 6

Se consideră *, Ryx ∈ pentru care numerele xyyx , şi 2222 )1()1( ++++ xxxxy sunt toate

numerele raţionale. Arătaţi că, de asemenea, x şi y sunt numere raţionale. (Profesor Nedeianu Dan, Concursul „Petre Sergescu”, 2008, clasa a VIII-a)

Soluţie: Din Qyx∈ şi Qxy∈ , rezultă Qx ∈2 şi Qy ∈2 . Al treilea număr este

QxyQyyxQxxy ∈+⇒∈+⇒∈++ )1()1( 222 , deci Qy∈ şi Qx∈ . În încheiere, vom prezenta o listă de probleme propuse, ce se pot aborda prin tehnicile etalate

anterior. 1) Fie xyx ,, numere reale nenule astfel încât zxyzxy ,, sunt numere raţionale.

a) Arătaţi că numărul 222 zyx ++ este raţional.

Page 23: RMM 8.pdf

CERCUL DE MATEMATICA

22- 22 -

b) Dacă, în plus, numărul 333 zyx ++ este raţional, să se arate că zyx ,, sunt numere raţionale.

(OJM 2001, clasa a VIII-a)

2) Fie ∗∈Qcba ,, astfel încât abccba=++

111 . Demonstraţi că numărul

)1)(1)(1( 222222 +++ cbcaba este raţional. (Profesor Nedeianu Dan, GM 4-5/1997)

*Această problemă a fost propusă şi la OLM Iaşi 2003, iar pentru z

cy

bx

a 1,1,1=== se

obţine problema 1 de la OJM 2002, clasa a VIII-a. 3) a) Fie x un număr real astfel încât xx +2 şi xx 23 + să fie raţionale. Arătaţi că x este număr raţional.

b) Arătaţi că există numere iraţionale x astfel încât xx +2 şi xx 23 − să fie raţionale. (OJM 2002, clasa a VIII-a)

4) Fie numerele reale distincte a şi b care au proprietăţile: Qba ∈+2 şi Qba ∈+ 2 . Arătaţi că:

a) Numerele 2

21+=a şi

221−

=b verifică enunţul.

b) Dacă }1{−∈+ Qba , atunci Qa∈ şi Qb∈ .

c) Dacă Qba∈ , atunci Qa∈ şi Qb∈ .

(OJM 2004, clasa a VIII-a) 5) Fie *,, Rcba ∈ astfel ca Qbcacab ∈,, . Să se arate că

22222 )())(( bcacabcbacba +++++++ este număr raţional. (Profesor Nedeianu Dan, RMT 3/2006)

6) Să se determine Nn∈ astfel încât Qn∈

++

200722 .

(Concursul “Traian Lalescu” 2007, clasa a VIII-a) BIBLIOGRAFIE [1] Matematica în concursurile şcolare, clasele V-VIII, Editura Paralela 45, 2001-2007; [2] Gazeta Matematică; [3] Revista de Matematică din Timişoara.

Page 24: RMM 8.pdf

CERCUL DE MATEMATICA

H SSM

- 23 -

Tema pentru grupa de performanta la clasa a- VIII -a

PRINCIPIUL INVARIANTULUI

Prof. Elena Rimnicianu Prof. Victor Saceanu

Scoala cu clasele I –VIII Nr. 11

In matematica “invariantul” este o marime, o relatie sau o proprietate ce ramane neschimbata in urma aplicarii sau interventiei unei transformari. Deci o situatie initiala este supusa in mod repetat unor transformari. De obicei se cere sa se demonstreze ca in urma acestor transformari se poate sau nu ajunge la o anumita forma. Aceasta se poate face alegand caracteristica obiectului care a fost supus transformarii, adica „invariantul” transformarii. Daca in final obiectul nu poseda “invariantul” atunci el nu poate fi obtinut in urma transformarilor descrise.

Aplicatii 1) Intr-un sistem cartezian xOy din punctul (x;y) este permisa deplasarea intr-unul din cele

patru puncte (x-2 ; y-2) ; (x-2 ; y+2) ; (x+2 ; y-2) ; (x+2 ; y+2). Demonstrati ca din punctul (0 ;0) nu se poate ajunge prin deplasari succesive in punctul (2008 ; 2009). Solutie Observam ca un punct (x ;y) avand suma coordonatelor x+y un numar par se

poate deplasa intr-un punct avand suma coordonatelor tot para, acesta fiind “invariantul ” I (x-2 ; y-2) are suma coordonatelor x+y-4- para II (x-2 ; y+2) are suma coordonatelor x+y- para III (x+2 ; y-2) are suma coordonatelor x+y –para IV (x+2 ; y+2) are suma coordonatelor x+y+4 – para Punctul (0;0) are suma coordonatelor 0, deci numar par, el se poate deplasa succesiv in puncte cu aceeasi paritate a sumei coordonatelor, deci nu se poate deplasa in punctul (2008;2009) care are suma coordonatelor impara.

2) Pe o tabla scriem numerele de la 1 la 2007. Printr-o operatie intelegem ca stergem doua numere oarecare de pe tabla si in locul lor scriem modulul diferentei celor doua numere. Sa se demonstreze ca dupa 2006 operatii pe tabla ramane un numar par. Solutie

La orice pas i, inainte de stergerea a doua numere a si b, fie Si+1 suma numerelor de pe tabla si Si suma numerelor de pe tabla dupa stergerea numerelor a si b. Atunci avem “invariantul ”:

Si+1- Si = a + b - ⏐a-b⏐= ⎩⎨⎧

≥<

badacabbadacaa

,2,2

. Deci Si+1- Si este totdeauna un numar par.

Suma 1+2+3+…+2007= 1004⋅2007 este un numar par, rezulta ca S1 este tot un numar par. Dar S1

este ultimul numar de pe tabla, deci pe tabla a ramas un numar par.

Page 25: RMM 8.pdf

CERCUL DE MATEMATICA

24- 24 -

3) Trei elevi sunt asezati in linie dreapta in ordinea X, Y, Z. Ei incep sa sara capra, adica unul peste altul (dar nu peste doi odata). Pot fi in aceasta ordine dupa 2009 sarituri ? Solutie

In urma unei sarituri, numarul elevilor inversati creste sau se micsoreaza cu 1, acesta fiind si “invariantul ”. Dupa un numar de 2009 sarituri (numar impar) va exista un numar impar de perechi de elevi inversati. Cum ordinea initiala nu contine o astfel de pereche, rezulta ca nu este posibil.

4) Consideram numerele a ; b + a ; b - a , a≥0. Dupa un pas inlocuim fiecare din numerele date prin media aritmetica a celorlalte doua. Se pot efectua mai multi pasi astfel incat sa obtinem numerele: a –1; b + 1+ a ; b+1 - a . Solutie

In acest caz ramane ca “invariant ” suma numerelor. Daca x, z,y sunt numerele, atunci ele se

inlocuiesc cu 2

;2

;2

yxxzzy +++ care au suma:

2)(2

222zyxzyzxyx ++

=+

++

++ = x+y+z

Suma numerelor initiale este: a+b+ a +b- a = a+2b, iar suma numerelor finale este: a-

1+b+1+ a +b+1- a = a +2b +1. Deci plecand de la numerele: a; b + a ; b - a nu se poate

ajunge la numerele a-1 ; b + 1+ a ; b+1 - a . BIBLIOGRAFIE : V. Pop, V. Lupsor – Matematica pentru grupele de performanta, clasele a VI-a, a VII-a.

Page 26: RMM 8.pdf

CERCUL DE MATEMATICA

H SSM

- 25 -

Tema pentru grupa de performanta la clasa a- IX -a Invarianţi

prof. Draga Tătucu Mariana elev Nistor Adriana Mihaela

Colegiul Naţional „Gheorghe Ţiţeica” Se întâmplă ca să întâlnim de multe ori probleme de combinatorică în care

avem o configuraţie iniţială, asupra căreia putem efectua un şir de transformări.

Se pune întrebarea dacă putem ajunge la o altă configuraţie dată cu ajutorul

acestor transforări. În astfel de probleme este utilă noţiunea de invariant, o

caracteristică care nu se schimbă în urma operaţiilor permise. Dacă această

caracteristică este diferită pentru configuraţia iniţială şi cea finală, răspunsul

este negativ. Spre exemplu: Pe tablă sunt scrise trei numere. La fiecare pas putem

aduna două din ele, să scădem din sumă al treilea număr, şi să scriem rezultatul

obţinut în locul celui de-al treilea număr. Dacă pe tablă erau 1,2,3 putem oare

să obţinem 10,20,30? Operaţia constă în transformarea tripletului (a,b,c) în (a,b,a+b-c). Trebuie deci să

căutăm o expresie (simetrică) în a,b,c astfel încât ea să nu se schimbe în urma

acestei operaţii, adică f(a,b,c)= f(a,b,a+b-c). Cunoaştem astfel de funcţii:

trinoamele de gradul doi. În cazul dat, dacă f(a,b,x)= x(a+b-x)+E, atunci f(a,b,c)= f(a,b,a+b-c). Deci f(a,b,c)= c(a,b,a+b-c) +E, unde E nu depinde de c.Acum găsim

uşor că f(a,b,c)= ab+bc+ca-a2 – b2 –c2. această expresie este invariantă în urma

operaţiei noastre. Cum f(1,2,3) = -3, iar f(10,20,30)= -300, problema este rezolvată. Alt tip de invarianţi pe mulţimi constă în colorarea unor elemente şi

sumarea unor cantităţi pentru anumite culori. De exemplu dacă efectuăm

operaţii pe o tablă de şah, putem considera suma unor cantităţă doar pe

câmpurile negre ale tablei. Spre exemplu: În vârfurile unei table 3x6 sunt plasate numerele de la dela 1 la 18. Este permis să alegem două câmpuri vecine pe

orizontală sau diagonală şi să micşorăm cu 1 numerele din ambele câmpuri.

Putem oare prin intermediul acestor operaţii să înlocuim numerele din fiecare

câmp cu opusele lor (adică x cu -x)? În această problemă colorăm tabla alb şi negru ca la şah. Considerăm D

diferenţa dintre suma numerelor de pe câmpurile albe şi suma numererlor de pe

câmpurile negre. Deoarece o pereche de câmpuri vecine conţine un câmp alb şi

unul negru, D este invariantă . Dacă am putea inlocui fiecare număr prin opusul

său, diferenţa ar deveni –D, deci acest lucru este posibil numai pentru D= -D, adică D= 0. Însă atunci suma numerelorde pe toată tabla ar fi pară, iar ea este

iniţial 1+2+3+4+........+18= 171 şi este impară. Vom expune câteva probleme care se rezolvă cu ajutorul invarianţilor: Exemplul 1:

Pe fiecare câmp al tablei de şah 8x8 avem câte un beculeţ. Iniţial toate

becurile sunt aprinse.Putem lua două câmpuri adiacente diagonal şi să

schimbăm starea lor. Este posibil ca după câteva operaţii toate becurile să fie

stinse, cu excepţia becurilor de pe câmpurile A1, A2? Indicaţie: Câmpurile adiacente diagonal au aceeaşi culoare. Deci paritatea

numărului de becuri aprinse de culoare albă este invariantă.

Page 27: RMM 8.pdf

CERCUL DE MATEMATICA

26- 26 -

Exemplul 2: Iniţal avem scris pe tablă numărul 4. La ficare pas, putem fie să ridicăm

numărul de pe tablă la pătrat, sau să-l micşorăm cu 3. Putem oare avea scris

numărul 2007 după câteva operaţii? Indicaţie: Restul numărului prin impărţirea la 3 este intotdeauna 1.

Exemplul 3: În vârfurile unui poligon regulat sunt scrise câteva numere. Iniţial doar

unul din ele este nenul. Putem alege câteva dintre vârfurile poligonului mare care să formeze un poligon regulat mai mic şi să adăugăm 1 la fiecare din numerele din aceste vârfuri. Este oare posibil ca după câteva operaţii toate numerele să

devină egale? Indicaţie: Dacă A1, A2, A3, .........., An sunt vârfurile poligonului, iar k1, k2, ......,

kn sunt numerele scrise în ele, vectorul k1OA1 + k2OA2 +..........+ knOAn este invariant, unde O este centrul poligonului.

Exemplul 4: Pe tablă sunt scrise trei numere. Putem efectua următoarea operaţie:

alegem unul din ele c şi-l înlocuim cu 2a + 2b –c, dacă a şi b sunt celelalte

numere. Este oare posibil să obţinem (5,13,42) din (1, 21, 42)? Indicaţie: Pentru această problemă invariantă este funcţia g(a,b,c)= 2ab+2bc+2ca

– a2-b2-c2.

Probleme propuse:

Problema 1: Fişele de la casino pot fi albe sau negre. Într-un rând sunt 10 fişe albe. La un pas putem scoate o fişă albă de pe tablă şi să schimbăm culoarea

fişelor vecine cu ea. Rămâne o singură fişă. Ce culoare are ea? Problema 2: Fie o tablă de şah 10x10. Pe câmpurile diagonalei principale se

află câte un pion. Avem voie să alegem doi spioni şi să-i mutăm cu câte un câmp

în sus. Putem oare să aducem cu astfel de opraţii toţi pionii pe o linie? Problema 3: Pe o tablă de şah sunt scrise numerele 1,2,3,...............,2001.

Putem înlocui două numere a şi b cu ab/(a+b+1). Repetăm operaţia aceasta până

pe tablă rămâne un singur număr. Care este acesta? Bibliografie: „Olimpiadele de matematică” - Iurie Boreico, Tudor Micu, Andrei Ciulpan, Laszlo Szilard - Ed.GIL2006

Page 28: RMM 8.pdf

CERCUL DE MATEMATICA

H SSM

- 27 -

Tema pentru grupa de performanta la clasa a-X-a

Câteva probleme generate de

exponenţiala de bază 2 Prof. Chirfot Carmen – Victoriţa

Colegiul Tehnic “Domnul Tudor”

Problema 1: Să se calculeze ],2[ori de apare2

....22

2

4434421n

respectiv *

ori de apare2

....22]2[

3

Nnn

∈43421 , unde ][x reprezintă

partea întreagă a lui x.

Soluţia 1:

221 << . Aplicând funcţia exponenţială de bază 2 ( este strict crescătoare ), obţinem

⇒<<221

222 22222222222

222222bazadeexp2<<⇒<<⇒<< .

Prin inducţie, se demonstrează că 1]2[221ori de apare2

....22

ori de apare2

....2222

=⇒<< 4342143421nn

. În mod analog,

demonstrăm că 1]2[ori de apare2

....22

3

=4434421n

.

Soluţia 2:

1]2[]2[22221ori1 de apare2

....22

ori de apare2

....22

ori1 de apare2

....22

ori1 de apare2

....22

ori de apare2

....22

32432

==⇒=<<<−−−44344214434421443442144344214434421

nnnnn

.

Problema 2: Să se calculeze *

ori de apare2

....22],2ln2[

2

Nnn

∈⋅4434421 .

Soluţie:

Deoarece 1ln2ln2ln202ln)2(ln1ln221ori de apare2

....22

ori1 de apare2

....22

ori1 de apare2

....22

222

=<<⋅<⇒<<⇒<<++

ennn443442144344214434421 .

Prin urmare, 0]2ln2[ori de apare2

....22

2

=⋅4434421n

.

Problema 3: Să se demonstreze că *

ori de apare2

....22

ori1 de apare2

....22,122

22

2

Nnnn

∈<−+

443442144 344 21 .

Page 29: RMM 8.pdf

CERCUL DE MATEMATICA

28- 28 -

Soluţie:

Deoarece ...112222221ori de apare2

....22

ori1 de apare2

....22

ori1 de apare2

....22

ori de apare2

....22

22

22

22

deqnnnn

⇒=−<−⇒<<<++

443442144 344 2144 344 214434421

Problema 4: Să se calculeze 4434421ori de apare2

....22

2

2limn

n ∞→.

Soluţie: Notăm 2lnln2 1ori1 de apare2

....221

2

nnn

n aaa =⇒= +

+

+ 4434421 . Evident, ( ) 1≥nna este strict crescător şi

mărginit. Conform teoremei lui Weierstress, şirul are limită finită. ( ) 1≥nna este mărginit deoarece

⇒<< 221 ⇒<<221

222 22222222222

222222bazadeexp2<<⇒<<⇒<< .

Prin inducţie, procedând analog, se demonstrează că )2,1(221ori de apare2

....22

2

∈⇒<< nn

a4434421 .

Prin trecere la limită în relaţia de recurenţă, găsim 2lnln ll = . Pentru rezolvarea ecuaţiei,

considerăm 2ln1)('2lnln)(,]2,1[: −=⇒−=→x

xfxxxfRf . Din 2ln

12ln10)(' =⇒=⇒= xx

xf .

Deoarece, ]2,1[2ln

12ln

12 ∉⇒< . Cum f’

strict pozitivă, avem f strict crescătoare.

Facând tabelul de semn, se observă că

unica rădăcină a funcţiei f inclusă în intervalul [1, 2] este 22 =⇒= lx .

Probleme propuse:

1. Să se demonstreze că *00 Nn ∈∃> ε astfel încât *

0 , Nnnn ∈≥ să

avem ε<−+

443442144 344 21ori de apare2

....22

ori1 de apare2

....22

22

2

22nn

.

2. Să se calculeze )22(limori de apare2

....22

2

−∞→ 4434421

nn

n .

x 1 2

f’(x) + + + + + + + + + + + + + + +

f(x) f(2) = 0

Page 30: RMM 8.pdf

CERCUL DE MATEMATICA

H SSM

- 29 -

Tema pentru grupa de performanta la clasa a- XII -a

Centralizatorul unui element într-un grup. Centrul unui grup. Teorema lui Lagrange.

Indicele unui subgrup într-un grup.

Prof.Bondoc Gabriela Roxana Grup Scolar Industrial Auto

Definiţia 1. Pentru x∈G vom nota CG(x) = { y∈G : xy=yx } care se numeşte centralizatorul lui x în G si Z(G)= I

Gx∈CG(x) care se numeşte centrul lui G.

In mod evident Z(G)= { x∈G ; xy = yx, pentru orice y∈G }. Propoziţia 2. Pentru orice x∈G, CG(x)≤G.

Demonstraţie. Dacă y, z ∈CG(x), atunci yx=xy şi zx=xz. Deducem că y-1x=xy-1 iar (y-1z)x=y-1(zx) = y-1(xz) = (y-1x)z = = (xy-1)z = x(y-1z), adică y-1z ∈ CG(x), deci CG(x) ≤G.

Corolar 3. Z(G)≤G. Demonstraţie. Avem Z(G)= I

Gx∈CG(x) şi Z(G)≤G.

Fie acum H≤G şi x∈G. Definim xH = { xh : h∈H } şi Hx = {hx : h∈H }. Mulţimea xH (Hx) poartă numele de

clasa la stânga (dreapta) a lui x în raport cu H. Propoziţia 4. Dacă x, y∈G şi H≤G atunci

(i) xH = yH ⇔ x-1y∈H (ii) Hx = Hy ⇔ xy-1∈H.

Demonstraţie. (i). Să presupunem că xH = yH. Cum 1∈H , x=x ⋅1 ∈ xH = yH, adică x = yh cu h ∈ H. Deducem că y-1x=h ∈H şi cum h-1 ∈H avem că h-1 = x-1y ∈ H. Reciproc, fie x-1y=h∈H şi z∈xH, adică z=xk cu k∈H. Cum x=yh-1 avem z=(yh-1)k = y(h-1k) , adică z∈yH (căci h-1k∈H ), deci xH⊆yH . Analog deducem că şi yH⊆ xH, de unde xH=yH.

(ii). Ca şi în cazul (i). Corolar 5. Dacă H≤G, atunci pentru x∈G, xH = H (sau Hx = H) ⇔ x∈H. În

particular, 1⋅H = H. Vom nota (G/H)s = {xH : x∈G} şi (G/H)d = {Hx : x∈G}

Propoziţia 6. (G/H)s şi (G/H)d sunt partiţii ale lui G. Demonstraţie. Este suficient să probăm pentru (G/H)s. Deoarece pentru orice x∈G avem x=x⋅1 ∈ xH deducem că U

Gx∈xH = G.

Fie acum x, y∈G şi să demonstrăm că xH=yH sau xH∩yH=Ø . Avem că x-1y∈H sau x-

1y∉H. Dacă x-1y ∈H, conform Propoziţiei 4, xH=yH. Să presupunem acum că x-1y∉H. Dacă ar exista z∈xH∩yH, atunci z=xh=yk cu h, k ∈H şi

am deduce imediat că x-1y= hk-1∈H -absurd. Deci în cazul x-1y∉H avem xH∩yH=Ø. Propoziţia 7. Funcţia f : (G/H)s →(G/H)d , f(xH)=Hx-1 pentru orice x∈G este o bijecţie.

Demonstraţie. Pentru x, y∈G echivalenţele xH=yH ⇔ x-1y∈H ⇔ x-1(y-1)-1 ∈H ⇔ Hx-1= Hy-1 (conform Propoziţiei 4) ne arată că f este corect definită şi că este injectivă. Cum surjectivitatea lui f este imediată, deducem că f este bijecţie.

Page 31: RMM 8.pdf

CERCUL DE MATEMATICA

30- 30 -

Din propoziţia precedentă deducem că |(G/H)s|= |(G/H)d|; acest număr cardinal se notează |G:H| şi poartă numele de indicele lui H în G.

Lema 8. Dacă H≤G şi x∈G, atunci |xH|=|Hx|=|H| . Demonstraţie. Este suficient să arătăm că mulţimile xH şi H sunt echipotente iar în acest sens definim fx : H →xH, fx(h) = xh pentru orice h∈H. Dacă h, k∈H şi fx(h) = fx(k) atunci xh=xk deci h=k adică f este injectivă. Cum fx este în mod evident şi surjectivă, deducem că fx este o bijecţie şi astfel |xH|=|H|.

Teorema 9. Dacă H≤G, atunci |G|= |H|⋅|G:H|.

Demonstraţie. Cum (G/H)s este o partiţie a lui G avem |G| = ∑∈Gx

|xH|(sumarea făcându-se după

clase distincte). Ţinând cont de Lema 8. deducem că |G|=|H|⋅|G:H|. În cazul în care G este un grup finit, atunci |G|, |H| şi |G : H| sunt numere naturale iar

relaţia |G|=|H|⋅|G:H|arată că |H| este un divizor al lui |G|. Teorema 10.(Lagrange) Ordinul oricărui subgrup al unui grup finit divide ordinul

grupului. Corolar 11. Dacă G este un grup finit de ordin n, atunci xn =1 pentru orice x ∈G.

Demonstraţie. Dacă k=o(x), atunci xk =1 şi k|n (conform teoremei lui Lagrange), adică n=kt cu t∈ℕ. Atunci xn=xkt=(xk)t=1t=1.

Aplicatii

1.Fie G un grup comutativ de ordin n. Arătaţi că produsul celor n elemente ale lui G este egal cu produsul elementelor de ordin cel mult 2. Aplicând acest rezultat grupului multiplicativ ),( * ⋅pZ , p prim să se demonstreze că 1)!1( +−pp . Rezolvare: Avem ∏∏∏

≤∈

>∈∈

⋅=

2)(2)(

)(

xoGx

xoGxGx

xxx şi vom demonstra că 1)(

2)(

=∏>

∈xoGx

x (*).

Dacă x G∈ cu o(x)>2 atunci 2)()( 1 >= −xoxo .Pe de altă parte 1−≠ xx (altfel x2=1). Deci în produsul (*) dacă apare x apare s inversul său deci 1)(

2)(

=∏>

∈xoGx

x , de unde concluzia.

Pentru a demonstra cea de a doua afirmaţie cunoscută sub numele de teorema lui Wilson, se ţine

cont de faptul că singurele elemente din *pZ de ordin mai mic sau egal cu 2 sunt 1̂ şi

∧−1p , deci

conform celor de mai înainte ^

1...2̂1̂ −⋅⋅⋅ p =∧−1p ⇒

^^1)!1( −=− pp ⇒ 1)!1( +−pp .

2. Fie n 2≥ un număr natural. Să se arate că n este un număr natural prim daca şi numai dacă orice grup cu n elemente are exact două subgrupuri. Rezolvare:Dacă n este un număr prim , considerând un grup arbitrar cu n elemente, orice subgrup al acestuia va avea ca ordin un divizor al lui n (Teorema Lagrange), adică pe 1 sau n. Rezultă că G are numai două subgrupuri şi anume {e} şi G.

Reciproc, să admitem că orice grup cu n elemente are numai două subgrupuri şi să arătăm că n prim. Presupunem prin reducere la absurd că n nu este prim şi fie atunci un divizor prim al său.

Să considerăm grupul aditiv cu n elemente Zn.. Subgrupul ciclic H generat de clasa ^/ pn are p

elemente şi cum 1<p<n rezultă H este subgrup al lui (Zn,+).Deci am găsit un grup cu n elemente care are trei subgrupuri , contradicţie cu ipoteza.

Page 32: RMM 8.pdf

CERCUL DE MATEMATICA

H SSM

- 31 -

3. Fie G un grup şi x un element de ordin finit din G. Dacă m şi n sunt doi întregi pozitivi cu proprietăţile: (n,m)=1, ord(xn)=m, ord(xm)=n, atunci ord(x)=nm. Rezolvare: Fie k=ord(x) Cum ord(xn)=m rezultă (xn)m =e adică xnm=e. De aici rezultă că .knmM (1) Cum xk=e, putem scrie xkm=e, adică (xm)k =e şi cum ord(xm)=n,rezulta .nk M (2) Analog se arată că

mk M .(3) Dar (n,m)=1 rezultă mnk .M (4) Din relaţiile (1) şi (4) rezultă k=mn.

4.Fie G un grup iar K G≤ .Să se demonstreze că HHZKCG )(,1)(1)( =⇔= a.î. GHK ≤≤ .

Rezolvare:Dacă alegem H a.î GHK ≤≤ , deoarece Z(H)≤CG(K)=1⇒Z(H)=1. Reciproc, presupunem că Z(H)=1 pentru orice H a.î. GHK ≤≤ . Dacă x∈CG(K) , atunci pentru H= }{xK ∪ avem GHK ≤≤ şi x∈Z(H)=1, deci CG(K)=1.

5.Să se demonstreze că pentru 3≥n ,Dn are un singur subgrup ciclic de ordin n. Rezolvare:Fie H= ρ nD≤ ; deoarece no =)(ρ deducem că H este un subgrup ciclic de ordin n.

Orice element din Dn\H este se forma ,ερ k k=0,1,…,n-1.De asemenea avem că

ερερερ 11 −− == n .Dacă presupunem că ερερ kk −= , atunci

ερερρερρερ )1(11 +−−−+ === kkkk prin urmare kk −= ρερ pentru orice k=0,1,...,n-1 şi

1)( 22 === − ερρεερρερ kkkkk . Rezultă că pentru orice HDx n \∈ avem o(x)=2. Dacă xH =′ , x∈Dn este un subgrup ciclic de ordinul n, atunci o(x)=n>2, deci x H∈ . Dar atunci HH ≤′ , nHH ==′ , deci .HH =′

BIBLIOGRAFIE 1. D. Buşneag : Teoria Grupurilor, Ed. Universitaria, Craiova, 1994 2. D.Buşneag : Capitole speciale de algebră,Ed. Universitaria,Craiova, 1997. 3. D. Buşneag, Fl. Chirteş, D. Piciu : Algebra, Ed. Universitaria, Craiova, 2001. 4. Al. Dincă : Lecţii de algebră, Ed. Universitaria, Craiova, 2000. 5. I. D. Ion, N. Radu : Algebra, Ed. Didactică şi Pedagogică, Bucureşti, 1991. 6. I. D. Ion, C. Năstăsescu, C. Niţă : Complemente de Algebră, Ed. Ştiinţifică şi Enciclopedică, Bucureşti,

1994. 7. D. Popescu, C. Vraciu : Elemente de teoria grupurilor finite, Ed. Ştiinţifică şi Enciclopedică, Bucureşti,

1986.

Page 33: RMM 8.pdf

CERCUL DE MATEMATICA

32- 32 -

Tema pentru grupa de performanta la clasa a- XII -a

ASUPRA ELEMENTELOR SIMETRIZABILE DINTR-UN MONOID

Prof.dr.Cainiceanu Gheorghe Prof.Leonard Giugiuc CNT

§1.Rezumat Tema se ocupa de prezentarea unor aspecte des folosite in aplicatii referitoare la functii injective si surjective ,introducerea notiunilor de elemente inversabile la stanga sau la dreapta intr-un monoid, rezultate de baza referitoare la acestea, si o lista de probleme rezolvate,respective propuse.

§2.Cateva rezultate generale Vom nota in cele ce urmeaza cu A o multime infinita si cu FA multimea tuturor functiilor definite pe A cu valori in A. Se presupun cunoscute notiunile de functie injectiva, surjectiva, bijectiva, inversabila. TEOREMA 1 Fie f,g : A→ A .Atunci avem:

1) Daca f si g sunt injective,atunci gf o este injectiva. 2) Daca f si g sunt surjective,atunci gf o este surjectiva. 3) Daca f si g sunt bijective,atunci gf o este bijectiva.

Demonstratie. 1) Fie x≠y. Deoarece functia g este injectiva avem g(x)≠g(y).Dar acum invocam injectivitatea lui f si deducem f(g(x))≠f(g(y)) adica ( gf o )(x) ≠( gf o )(y). 2) Pentru un z din A privit codomeniu pentru f exista y din A codomeniu pentru g astfel incat

zyf =)( .Acum exista x din A,domeniul lui g pentru care yxg =)( . In concluzie, .))((.. zxgfiAaxAz =∈∃∈ 3)Este o consecinta pentru 1) si 2). TEOREMA 2 O functie AFf ∈ este inversabila daca si numai daca este bijectiva. TEOREMA 3 Daca pentru AFgf ∈, avem gf o bijectiva, atunci g este injectiva iar f surjectiva. Demonstratie. Vom demonstra ca g este injectiva. Daca )()())(())(())(())((,, ygxgygfxgfygfxgfyxAyx ≠⇒≠⇒≠⇒≠∈ oo Vom demonstra ca f este surjectiva. Fie Az∈ .Atunci .))(())(( zxgfzxgfAx =⇒=∈∃ o Notam zyfyxg =⇒= )()( .Am aratat ca: zyfAyAz =∈∃∈ )(| ,deci f este surjectiva. TEOREMA 4 Fie AFf ∈ o functie injectiva si nesurjectiva. Atunci exista AFg ∈ o functie neinjectiva si surjectiva astfel incat Afg 1=o . Demonstratie. Daca f este injectiva, functia )()(,Im: 11 xfxffAf =→ este o bijectie.deci are

inverse Aff →− Im:11 .Definim functia

⎩⎨⎧

∈∈

=→−

fAxdacaafxdacaxf

xgAAgIm\,

Im,)()(,:

11

Unde a este un numar arbitrar fixat din A.Evident sunt adevarate urmatoarele xxffxfgxfgAx ===∈ − ))(())(())((, 1

11 g este surjectiva (imaginea este egala cu codomeniul). Desigur g nu este injectiva deoarece aygxgiafAyfx ==∈∃∈∃ )()(..,Im\,Im . TEOREMA 5 Fie AFg ∈ o functie neinjectiva si surjectiva. Atunci exista o functie AFf ∈ injectiva si nesurjectiva astfel incat Afg 1=o . Demonstartie. Pentru orice x∈A avem Φ≠− })({1 xg ,pentru ca g este surjectiva.

Page 34: RMM 8.pdf

CERCUL DE MATEMATICA

H SSM

- 33 -

Cum g este neinjectiva exista cel putin un x∈A astfel incat multimea })({1 xg − are ce putin doua elemente. Pentru fiecare x∈A alegem un unic yx∈ })({1 xg − . Notam B1=U

Axxy

}{ . Evident, BB ⊂1 si incluziunea este stricta,iar )()(,: 111 xx ygygABg =→ este

o bijectie.Deci exista inverse sa. Fie acum xygxggxggAx x ===⇒∈ −− )())(())(( 11

11o .

In concluzie, 11−g este functia cautata.

APLICATIE Fie ⎪⎩

⎪⎨⎧

∈−=→ QRx

x

QxxxfRRf \1)(,: .Sa se arate ca f este bijectiva si sa se

determine inverse sa.

Solutie: )(1\,

,\,

)(1

),(

\)(,)(

1)(,)(

))(())(( xQRxxQxx

QRxxf

Qxxf

QRxfxf

Qxfxfxffxff R=

⎩⎨⎧

∈∈

=⎪⎩

⎪⎨⎧

∈−=

⎪⎩

⎪⎨⎧

∈−==o

Cum 1R bijectiva deducem ca f este injective si surjectiva (TEOREMA 3). Deci f este inversabila si prin compunere cu inversa deducem ff =−1 .

§3Elemente simetrizabile stanga,dreapta In monoidul ( ),oF vom considera functia f surjectiva si neinjectiva. TEOREMA 6 Functia f este simetrizabila la dreapta, dar nu este simetrizabila la stanga. Demonstratie. Din TEOREMA 5 ,avem ca exista o functie AAg →: ,injective si nesurjectiva cu proprietatea ca Rgf 1=o ,deci f este inversabila la dreapta. Presupunand prin absurd ca f ar fi simetrizabila si la stanga am contrazice Teorema 3. TEOREMA 7 Daca f este o functie injective si nesurjectiva atunci in monoidul ( ),oF ,ea este un element simetrizabil la stanga si nesimetrizabil la dreapta. Demonstratie. Din Teorema 4 deducem ca exista AAg →: surjectiva si neinjectiva cu proprietatea Afg 1=o deci f e simetrizabila la stanga. Daca f ar fi simetrizabila la dreapta,din Teorema 3 avem ca f este surjectiva,contradictie. COMENTARIU. Considerentele de mai sus arata necesitatea imperioasa ca atunci cand determinam elemental simetric al unui element dintr-o structura sa verificam exyyx =∗=∗ .

§4 Probleme de rezolvat 1. Consideram ,,...2,,..),(, nBABABAAiaZMBA n +++∈ inversabile in )(ZM n .

Demonstrati ca BkA )12( −+ este inversabila in )(ZM n .

2. Se considera inelul (A,+,·), cu elemental unitate 1.Aratati ca daca 1+xy este inversabil,atunci si 1+yx este inversabil, unde x,y, sunt elemente fixate in inel.

3. Fie ABBAcuRMBA n =+∈ ,),(, .Demonstrati ca: a) BIAI nn −=− −1)( b) BAAB = .

4. Pe multimea M se considera o lege decompozitie asociativa notata multiplicativ. Se

presupune ca exista a∈M cu proprietatea ca }|{, MxaxaamayMy ∈=∈∈ Sa se arate ca legea de compozitie admite un element neutru.

Bibliografie [1]C.Nastasescu,C.Joita,M.Brandiburu,D.Joita - Culegere de exercitii de algebra pentru liceu, Bucuresti,1996 [2]I.D.Ion,A.Ghioca,N.Nedita - Matematica,clasa a XII-a,EDP 1983 [3]C.Nastasescu,C.Nita,C.Vraciu - Bazele Algebrei,VolI, Editura Academiei Romane-Bucuresti’86

Page 35: RMM 8.pdf

CERCUL DE MATEMATICA

34- 34 -

Tema pentru grupa de performanta la clasa a- XII -a

Extinderi de inele şi corpuri (I).

prof. Adrian Lupu, Dr. Tr. Severin În cele ce urmează ne propunem să trecem în revistă câteva elemente legate de extinderile de inele şi corpuri, prin câteva grupe de exemple considerate relevante, unele completate şi cu mici suplimente teoretice, accesibile elevilor de clasa a XII-a şi foarte utile şi în alte aplicaţii. Facem precizarea că vom viza noţiunea de inel introdusă mai general, în sensul că nu mai impunem condiţia ca şi legea a doua să aibă element neutru. Dacă acest lucru s-ar întâmpla am avea cazul particular al inelului unitar, din manualul de algebră antedecembrist. Această manieră de abordare va apărea oricum la nivel universitar, dar este preluată şi în unele din recentele manuale alternative sau culegeri de liceu (de exemplu cel de Mircea Ganga). Restul noţiunilor legate de substructurile şi extinderile de structuri din titlu se introduce similar. Pentru ca elevii să conştientizeze ce diferenţe pot apărea din această abordare propunem pentru început parcurgerea următoarelor exerciţii:

a) Să se arate că

QNnaaZ n ⊂⎭⎬⎫

⎩⎨⎧ ∈∈=⎥⎦

⎤⎢⎣⎡ ,,

221 Z

este subinel al lui ( )⋅+,,Q b) Arătaţi că singurul subinel unitar al lui Z este Z însuşi (via subgrupurile care sunt de

forma nZ şi, fiind unitar, 11 =⇒∈ nZ )

Obs. ⎥⎦⎤

⎢⎣⎡

21Z este cel mai mic subinel al lui Q care conţine Z şi

21 .

Generalizare: Dacă p este un număr prim atunci

QNnapa

pZ n ⊂

⎭⎬⎫

⎩⎨⎧

∈∈=⎥⎦

⎤⎢⎣

⎡,,1 Z este subinel al lui ( )⋅+,,Q .

c) [ ] { }Z∈++= cbacbaZ ,,|422 333 , cu adunarea şi înmulţirea obişnuite , este subinel în ( )⋅+,,R .

d) ⎭⎬⎫

⎩⎨⎧

∈∈⎟⎟⎠

⎞⎜⎜⎝

⎛== RxQa

axa

M xa ,|0,A este subinel comutativ şi unitar în raport cu adunarea şi

înmulţirea matricilor din ( )R2M . e) Arătaţi că următoarele mulţimi de numere, împreună cu operaţiile obişnuite de adunare şi

înmulţire sunt subinele ale lui ( )⋅+,,C , precizându-le pe cele unitare. 1) 2Z (numerele întregi, pare) 2)mZ , unde Z∈m (deci multiplii întregi ai lui m) 3) { }Z∈+ baba ,|3 4) { }Z2,|3 ∈+ baba 5) { }Qbaba ∈+ ,|3 6) { }Qbabia ∈+ ,| - inelul întregilor lui Gauss 7) { }Z3,| ∈+ babia 8) { }Qbabia ∈+ ,|

9) { }Z∈+ baiba ,|3 10) ⎭⎬⎫

⎩⎨⎧

∈+ paritate aceeasiau b si a,|

23 Z,,babia

11) { }QdcbadcbaxRx ∈+++=∈ ,,,,632, f) Arătaţi că următoarele mulţimi de matrice formează subinele ale inelelor de matrice

pătratice corespunzătoare, împreună cu operaţiile uzuale de adunare şi înmulţire, precizând care dintre aceste subinele sunt unitare şi, respectiv, care sunt comutative:

Page 36: RMM 8.pdf

CERCUL DE MATEMATICA

H SSM

- 35 -

1) ⎭⎬⎫

⎩⎨⎧

∈⎟⎟⎠

⎞⎜⎜⎝

⎛ Zbaabba

,,3

2) ⎭⎬⎫

⎩⎨⎧

∈⎟⎟⎠

⎞⎜⎜⎝

⎛Zba

abba

2,,3

3) ⎭⎬⎫

⎩⎨⎧

∈⎟⎟⎠

⎞⎜⎜⎝

⎛Qba

abba

,,3 4)

⎭⎬⎫

⎩⎨⎧

∈⎟⎟⎠

⎞⎜⎜⎝

⎛ −Zba

abba

,,

5) ⎭⎬⎫

⎩⎨⎧

∈⎟⎟⎠

⎞⎜⎜⎝

⎛ −Zba

abba

3,, 6) ⎭⎬⎫

⎩⎨⎧

∈⎟⎟⎠

⎞⎜⎜⎝

⎛ −Qba

abba

,,

7) ⎭⎬⎫

⎩⎨⎧

∈⎟⎟⎠

⎞⎜⎜⎝

⎛ Zbaabba

,| 8) ⎭⎬⎫

⎩⎨⎧

∈⎟⎟⎠

⎞⎜⎜⎝

⎛ Zbab

a,|

00

9) ⎪⎭

⎪⎬

⎪⎩

⎪⎨

⎧∈

⎟⎟⎟

⎜⎜⎜

⎛Zcba

abacba

,,|00

0 10) ⎪⎭

⎪⎬

⎪⎩

⎪⎨

⎧∈

⎟⎟⎟

⎜⎜⎜

⎛Rx

xx

xx|

0000

0

11) ⎪⎭

⎪⎬

⎪⎩

⎪⎨

⎧∈

⎟⎟⎟

⎜⎜⎜

⎛Rcba

cb

a,,|

000000

12) ⎭⎬⎫

⎩⎨⎧

∈⎟⎟⎠

⎞⎜⎜⎝

⎛ Zaa

|000

13) ⎭⎬⎫

⎩⎨⎧

∈⎟⎟⎠

⎞⎜⎜⎝

⎛ Zaa

|0002

g) Arătaţi că fiecare din următoarele mulţimi de funcţii reale definite pe [ ]1,1− , împreună cu operaţiile obişnuite de adunare şi înmulţire, formează subinele unitare şi comutative ale inelului funcţiilor definite pe [ ]1,1− :

1) mulţimea funcţiilor continue 2) mulţimea funcţiilor pare 3) mulţimea funcţiilor polinomiale 4) mulţimea funcţiilor derivabile 5) mulţimea funcţiilor mărginite. Rafinarea discuţiei de la inel la corp presupune automat ca inelul să fie unitar, în legătură cu

simetrizabilitatea oricărui element nenul în raport cu a doua lege de compoziţie. Prezentăm mai detaliat două exemple speciale de corpuri. 1) Corpul cuaternionilor, introdus de R. Hamilton (1854), obţinut după introducerea a

două entităţi matematice j, k asemănătoare unităţii imaginare i, din C, pentru care prelungim înmulţirea cu regulile: jkiikikjjkkjiijkji =−==−==−=−=== ,,,1222 , reguli uşor de reţinut dacă avem în vedere permutările circulare ale elementelor i, j, k în această ordine. Un cuaternion este un număr de forma dkcjbia +++ , unde Rdcba ∈,,, . Pe mulţimea H a cuaternionilor, care extinde C, definim adunarea şi înmulţirea elementelor

kdjcibaqkdjcibaq 2222211111 , +++=+++= : ( ) ( ) ( )kddjccibbaaqq 2121212121 +++++++=+

( ) ( ) +−+++−−−=⋅ idcdcbabaddccbbaaqq 122112212121212121 ( ) ( )kdacbcbdajdbcadbca 1212212112122121 +−++++−+

(în esenţă sunt regulile de calcul cu expresii algebrice suplimentate cu cele de mai sus). E util să prezentăm şi extinderile noţiunilor de conjugat şi normă. Mai precis, pentru

Hdkcjbiaq ∈+++= atunci conjugatul său este Hdkcjbiaq ∈−−−= şi funcţia normă: 2222)(,: dcbaqqqNRHN +++=⋅=→ + .

E uşor de verificat că ( )⋅+,,H este inel unitar, cu elementele neutre: kjikji ⋅+⋅+⋅+=⋅+⋅+⋅+= 00011,00000 . Pentru a arăta inversabilitatea oricărui

Page 37: RMM 8.pdf

CERCUL DE MATEMATICA

36- 36 -

dkcjbiaqqHq +++=≠∈ ,0, (deci 02222 >+++ dcba ) uzităm de : 1)()(

=⋅=⋅ qqN

qqN

qq deci q

este inversabil şi chiar putem explicita: kqN

djqN

ciqN

bqN

aqN

qq ⋅−⋅−⋅−==−

)()()()()(1 cu Hq ∈−1 .

Se verifică uşor faptul că acest corp nu este comutativ. 2) Dacă { }1\Nd ∈ este un întreg liber de pătrate (i.e. în descompunerea lui d în factori

primi nu apare niciun pătrat perfect), cu adunarea şi înmulţirea uzuale, mulţimea

[ ] { }QbadbadQnot

∈+= ,, este un corp comutativ, numit corp pătratic, iar numerele de forma dba + , cu d întreg liber de pătrate şi Qba ∈, se numesc numere pătratice. Pentru numărul

[ ]dQdbaz ∈+= definim previzibil conjugatul său pătratic: dbaz −= . Raţiunea apariţiei unor astfel de corpuri e dată de rezolvarea, de exemplu, în [ ]3Q a

ecuaţiei 32 =x pentru că în Q ea nu avea soluţii. Considerăm utilă pentru elevii interesaţi de tema propusă şi prezentarea câtorva alte

exemple de subcorpuri, în raport cu operaţiile uzuale de adunare şi înmulţire: 1) { }Qbaba ∈+ ,,3 2) { }Qbabia ∈+ ,,

3) { }Qbaiba ∈+ ,,3 4) ⎪⎭

⎪⎬⎫

⎪⎩

⎪⎨⎧

∈+−

=+ Rbaiba ,,2

31,εε

5) { }Qcbacba ∈++ ,,,42 33 6) ⎭⎬⎫

⎩⎨⎧

∈⎟⎟⎠

⎞⎜⎜⎝

⎛ −Qba

abba

,|

7) ⎭⎬⎫

⎩⎨⎧

∈⎟⎟⎠

⎞⎜⎜⎝

⎛ −Qba

abba

,|3 8)

⎭⎬⎫

⎩⎨⎧

∈⎟⎟⎠

⎞⎜⎜⎝

⎛−−

+Qba

babbba

,|4

9) ⎭⎬⎫

⎩⎨⎧

∈⎟⎟⎠

⎞⎜⎜⎝

⎛−

Qbaabba

,| 10)

⎪⎪⎭

⎪⎪⎬

⎪⎪⎩

⎪⎪⎨

⎟⎟⎟⎟⎟

⎜⎜⎜⎜⎜

−−

−−−−

Qdcba

abcdbadc

cdabdcba

,,,|

Pentru final selectăm câteva din problemele înrudite cu tema anunţată, propuse la diverse concursuri de matematică şi nu numai:

I) Fie QC \∈α cu proprietatea că mulţimea { }Z∈+= babaA ,,α este inel faţă de operaţiile uzuale din C. Ştiind că inelul are exact 4 elemente inversabile, să se arate că [ ]iZA = .

(Olimpiada Naţională – Suceava 1997) II) Se consideră un inel A. a) Să se arate că mulţimea { }AxxaaxAaA ∈=∈= oricepentru ,|)(Z este subinel al

inelului A. b) Să se arate că dacă orice subinel comutativ al lui A este corp, atunci A este corp.

(Olimpiada Naţională – Vâlcea 2002) III) Inelul întregilor pătratici. Fie d un număr întreg liber de pătrate; definim [ ] { }ZnmdnmxCxdZ ∈+=∈= ,,| . Atunci: 1) [ ]( )⋅+,;dZ este un subinel al corpului numerelor complexe, chiar domeniu de integritate.

2) [ ]dZ este izomorf cu inelul matricelor de forma Znmmdnnm

∈⎟⎟⎠

⎞⎜⎜⎝

⎛,, , în raport cu

operaţiile uzuale cu matrice. 3) Inelele [ ]dZ şi [ ]'dZ sunt izomorfe dacă şi numai dacă 'dd = (un astfel de

izomorfism f între cele două inele invariază elementele lui Z şi atunci el este bine determinat de valoarea ( )df ).

Page 38: RMM 8.pdf

CERCUL DE MATEMATICA

H SSM

- 37 -

4) Subinelele unitare ale lui [ ]dZ sunt de forma { } NnZbadbnaAn ∈∈+= ,,| 5) Definim aplicaţia normă [ ] ( ) 22,: dnmdnmNZdZN −=+→ . Dacă notăm cu

dnmx −= conjugatul întregului pătratic dnmx += , se arată că N are proprietăţi asemănătoare modulului: )()()(,)( yNxNxyNxxxN ⋅=⋅= . De aici: [ ]( ) { }1)()( ±=∈⇔∈ ZUxNdZUx .

6) Grupul multiplicativ ale elementelor inversabile din [ ]iZ este [ ]( ) { }iiZU ±±= ,1 . 7) Dacă { }5,3,2∈d , atunci [ ]( )dZU conţine o infinitate de elemente şi putem găsi în [ ]( )dZU

elemente pozitive oricât de mici (este suficient să găsim un singur element, considerând apoi puterile acestuia şi conjugatele lor).

Problemele 1-6 sunt rezolvate în [3], iar 7 poate fi găsită în variantele examenului de bacalaureat din ultimii ani.

IV) Legat de corpul numerelor pătratice mai inserăm şi : 1) [ ]( )⋅+,;dQ este subcorp al lui C (inversul elementului nenul dba + este

( ) ( )dQdbadba

∈−− 22

1 , deoarece 022 ≠− dba , altfel se ajunge la QRbad \∉±= ).

2) ( )dQ este izomorf cu mulţimea matricelor de forma Qbaadbba

∈⎟⎟⎠

⎞⎜⎜⎝

⎛,, , care formează

corp în raport cu operaţiile uzuale de adunare şi înmulţire. 3) Corpurile ( )dQ şi ( )'dQ sunt izomorfe dacă şi numai dacă 'dd = ; singurele

automorfisme ale corpului ( )dQ sunt aplicaţia identică şi cea de conjugare, ambele invariind elementele lui Q.

4) Dacă un subcorp CK ⊂ este astfel încât { }gfEndK ,= şi Qxxgxf ∈⇒= )()( , atunci există un întreg liber de pătrate 1≠d pentru care ( )dQK = .

Olimpiada de Matematică Naţională, 1988 , Marcel Ţena Problemele 1-3 pot fi găsite în [3]. Într-un viitor articol ne propunem să sintetizăm câteva elemente teoretice suplimentare

legate de aceeaşi tematică , esenţiale în fundamentarea ştiinţifică a rezolvării unor probleme mai speciale de construcţii geometrice cu rigla şi compasul, de exemplu cele trei probleme celebre ale antichităţii.

Bibliografie: 1.M.Ţena –Algebra.Structuri fundamentale pentru liceu , Ed. Corint ,Bucureşti , 1996 2.Gh. Ekcstein –Olimpiadele şi concursurile pentru clasele IX-XII , anii 2001-2008, Ed.

Bârchi , Timişoara 3.C. Niţă , T. Spircu –Probleme de structuri algebrice , Ed. Tehnică, Bucureşti , 1974 4.I. Tofan , C. Volf – Algebră-Inele , module , Teorie Galois , Ed. Matrix-Rom ,

Bucureşti , 2001 5.M . Ganga –Manual de clasa a XII a , Ed. Mathpress, Ploieşti , 2001 6.V. Schneider –Probleme de algebră pentru clasa a XII a , Ed. Valeriu , Craiova ,

1996

Page 39: RMM 8.pdf

PROBLEME PROPUSE

- 36 -

Elevii vor rezolva probleme de la clasa pe care o urmează şi de la clasa inferioară. Se pot rezolva şi problemele date la Ediţia a IV-a a concursului „Petre Sergescu” - pagina 3. Soluţiile redactate pe foi format A5 se vor preda profesorului îndrumător. IV.1. În 8 cutii se repartizează în mod egal 48 de bile roşii şi albastre. Care este

numărul minim de bile albastre care trebuie să existe, astfel încât oricum ar fi repartizate bilele în cutii, în fiecare cutie să se găsească cel puţin o bilă albă?

Inst. Niculina Oprita IV.2. Ionuţ îşi propune să rezolve un anumit număr de probleme. Dacă ar rezolva

cate 7 probleme pe zi le-ar termina într-un anumit număr de zile,însă el rezolvă câte 10 probleme pe zi şi termină cu 3 zile mai devreme. Câte probleme a avut de rezolvat Ionuţ?

Inst. Maria Ungureanu

V.1. Sa se afle numerele naturale x,y, z astfel incat : xy⋅z- 20090 = 23⋅251. prof. Elena Rimnicianu

V.2. Scrieti numarul 2008

1200910 − ca suma de zece puteri cu aceeasi baza.

prof. Elena Rimnicianu

V.3. Sa se arate ca oricare ar fi n∈N, numarul 3535 83 ++ +

nn

nu este patrat perfect. prof. Elena Rimnicianu

V.4. Poate fi numarul x4k+10y+2 patrat perfect oricare ar fi x, y, k∈N? prof. Elena Rimnicianu

V.5. Să se scrie numărul 5 3• 2008 ca o diferenţă de două cuburi perfecte. Prof. Nedeianu Dan

V.6. Să se rezolve în N ecuaţia: 2x+2y+2z = 4672, unde x<y<z Prof. Ionică Constantin

V.7. Să se determine cifra x astfel ca numărul 0876x să se dividă cu 31. Prof. Ionică Constantin

V.8. Să se determine x, y∈N ştiind că x2y + xy = 272. Prof. Ionică Constantin

V.9. Dacă x,y sunt numere naturale cu x-y<2008, iar câtul şi restul împărţiri lui x la y sunt 201, respectiv 8, să se afle numerele x si y.

Prof. Nedeianu Dan V.10. Varstele tatalui, fiului si nepotului sunt exprimate prin numere prime, iar peste

5 ani varstele lor vor fi exprimate prin numere naturale patrate perfecte. Cati ani are fiecare?

prof. Osain Victoria V.11. Determinati numerele naturale x, y, z pentru care xy+yz+zx = 11 si x≤y≤ z.

prof. Osain Victoria V.12. Aflati numarul prim “p”, stiind ca p2-2, 2p2-1, 3p2 +4 sunt de asemenea prime.

prof. Osain Victoria

Page 40: RMM 8.pdf

PROBLEME PROPUSE

H SSM

- 37 -

VI.1. Fie n∈N. Sa se arate ca numarul A = 22n (26n + 29n ) - 3n (5n+1) este divizibil cu 101.

prof. Elena Rimnicianu VI.2. Demonstrati ca daca n∈N si 11|4n +5 atunci 11|21n2+22n +5.

prof. Elena Rimnicianu VI.3. Daca n∈N, n impar, a =2p, p∈N si k∈N, demonstrati ca fractia

annananaa

k

kkk

++++⋅+ )(23

este ireductibila.

prof. Elena Rimnicianu VI.4. În triunghiul ABC cu m(Ĉ)=75 o , înalţimea [AD] este jumătate din latura [BC].

Să se afle m(Â). Prof. Nedeianu Dan

VI.5. Aflaţi toate fracţiile ordinare cu numitorul 185 cuprinse între 41 şi

31 şi al căror

numărător este divizibil cu 7. Prof. Ionică Constantin

VI.6. În ∆ABC avem: m( A) = 4x + 100 şi m( B) = 6x - 600. Se cere: a) Între ce limite poate varia x astfel ca ∆ABC să existe? b) Dacă ∆ABC este dreptunghic să se determine x şi unghiurile ∆ABC. c) Să se afle valorile lui x pentru care ∆ABC este isoscel.

Prof. Ionică Constantin VI.7. Să se determine numerele naturale x, y, z distincte două cate două pentru care

xyz + xy + xz + yz + x + y + z = 2008. Prof. Nedeianu Dan

VI.8. Intr-un patrulater convex triunghiurile formate de punctual de intersectie al diagonalelor cu varfurile au acelasi perimetru. Sa se arate ca patrulaterul este romb.

prof. Osain Victoria VI.9. Sa se demonstreze ca din cifrele 1, 2, 3, 4, 5, 6, 7 nu se pot forma doua

numere de cate sapte cifre, diferite ( in fiecare numar fiecare cifra intra o singura data) astfel incat unul sa se divida cu celalalt.

prof. Osain Victoria

VII.1. Să se calculeze valoarea numărului 9080...5040302010 , unde indicii reprezintă baze de

numeraţie. Evident, calculul se face de jos în sus. Prof. Chirfot Carmen-Victoriţa

VII.2. Sa se afle numerele intregi x,y,z stiind ca : x2 –x + 3−y + (z2 –9)2 ≤0. prof. Elena Rimnicianu

VII.3. Triunghiul ABC are m(<ABC) = 2m(<ACB). Prin varful A ducem o paralela la bisectoarea unghiului ABC, care intalneste pe BC in M. Daca (AB este bisectoarea unghiului MAC stabiliti natura triunghiului ABC.

prof. Elena Rimnicianu

Page 41: RMM 8.pdf

PROBLEME PROPUSE

- 38 -

VII.4. Rezolvati ecuatia .2}]{[

1}{

1][

2=−+

xxxx

([x] si {x} reprezinta partea intreaga, respectiv partea fractionara a lui x). prof. Elena Rimnicianu

VII.5. Sa se rezolve in R ecuatia 9{x}2-6x +1=0 , ({x} - partea fractionara a lui x). prof. Elena Rimnicianu

VII.6. Determinati numerele reale a, b, c astfel incat

a+b+c = 3k si ab +ac +bc = 3k2, unde k ∈R. prof. Elena Rimnicianu

VII.7. Ipotenuza BC a triunghiului dreptunghic ABC este împarţită de punctele M si N

în trei părţi egale. Calculaţi valoarea maximă a raportului BC

ANAM + .

Prof. Nedeianu Dan VII.8. Să se demonstreze că numărul 2005·2006·2007·2008 + 1 este pătrat perfect.

Prof. Ionică Constantin VII.9. Să se determine n ∈ N pentru care 2+n + 53 −n = 7

Prof. Ionică Constantin VII.10. În triunghiul ABC construim linia mijlocie DE, D∈[AB], E∈[AC] şi G

centrul de greutate al triunghiului BCD, BG ∩AC={F} . Calculaţi raportul GFBG .

Prof. Nedeianu Dan VII.11. Sa se arate ca ecuatia x2 +y2 =2007 nu are solutii numere intregi.

prof. Osain Victoria

VIII.1. Sa se arate ca 3⏐a3+b3 daca si numai daca 3 ⏐a+b. prof. Elena Rimnicianu

VIII.2. Sa se rezolve in N ecuatia : 35

77

66

5=⎥⎦

⎤⎢⎣⎡ +

+⎥⎦⎤

⎢⎣⎡ +

+⎥⎦⎤

⎢⎣⎡ + xxx , unde [x] reprezinta

partea inteaga a lui x. prof. Elena Rimnicianu

VIII.3. Daca a, b, c ∈[0,1] cu ab + ac + bc = 1, demonstrati ca an +bn +cn≤2, oricare ar fi n∈N*.

prof. Elena Rimnicianu VIII.4. Fie ABCD un pătrat de latura a si O punct oarecare în interior. Pe

perpendiculara in O pe planul (ABCD) se ia un punct V, cu OV=2a . Să se arate ca

suma pătratelor distanţelor de la V la laturile pătratului este mai mare sau egală cu 2a 2 .

Prof. Nedeianu Dan VIII.5. Ştiind că o prismă are 550 diagonale, să se determine numărul laturilor

poligonului convex de la baza ei. Prof. Ionică Constantin

VIII.6. Să se arate că 962 2 ++ xx + 4422 22 +−−+ xxyyx ≥ 5, x,y ∈ R. Prof. Ionică Constantin

Page 42: RMM 8.pdf

PROBLEME PROPUSE

H SSM

- 39 -

VIII.7. Să se rezolve în numere întregi ecuaţia: 22 )2()6( +++ yx + 22 )3()9( −+− yx = 5 10

Prof. Ionică Constantin

VIII.8. Să se calculeze raportul ba , ştiind că a, b∈R ∗ şi că a+2008 ab =2009b.

Prof. Nedeianu Dan VIII.9. Sa se arate ca ecuatia 3x2 - 4xy + 2y2 - 21x + 12y – 3 = 0 nu are solutii intregi.

prof. Osain Victoria VIII.10. Sa se arate ca ecuatia 2x +7y =19z nu are solutii numere naturale.

prof. Osain Victoria

IX.1. Dacă x2+y2+z2+t2+(x+y+z+t)2 = 5 , cu x,y,z,t ∈R, să se arate că 2,2,2,2 ≤≤≤≤ tzyx

Prof. Nedeianu Dan

IX.2. Să se rezolve sistemul

[ ] { }[ ] { }[ ] { }⎪

⎪⎨

=++=++=++

3,12,11,0

yxzxzyzyx

Prof.Daniel Sitaru IX.3. Sa se arate ca pentru orice numar natural n, exista un numar natural K avand

n cifre numai 1 si 2, cu proprietatea ca 2n divide pe K. Prof. Cainiceanu George

IX.4. Să se rezolve în mulţimea numerelor reale sistemul de ecuaţii: ⎪⎩

⎪⎨

=++=−=+

6273

2

2

zyxxyyxyx

Prof. Grecu Vasile – Colegiul Economic IX.5. Să se determine numerele reale strict pozitive care îndeplinesc condiţiile:

1222

,1222

,1222

≤++

+≤

++

+≤

++

+

zxyzxy

xz

zxyzxy

zy

zxyzxy

yx

Prof. Grecu Vasile – Colegiul Economic IX.6. Să se rezolve în R

32005

220062007-x

20053

20062

20071 −

+−

+=−

+−

+− xxxxx

prof. Daniel Stretcu – Colegiul Naţional “Gh. Ţiţeica” IX.7. Dacă nb este o progresie geometrică cu termeni scrict pozitivi, să se arate că

1),1)(( 121 ≥−−+≥ nnbbbbn . Prof. Nedeianu Dan

IX.8. Să se determine cea mai mare valoare pentru S = x1x2 + x2x3 + … + +x2008x2009, ştiind că x1, x2 … x2009 sunt numere nenegative cu proprietatea x1+ x2 + x3 + … + x2009 = 1

prof. Daniel Stretcu – Colegiul Naţional “Gh. Ţiţeica”

IX.9. Rezolvati ⎭⎬⎫

⎩⎨⎧ +

=⎭⎬⎫

⎩⎨⎧ −

31

21 xx (notam { }x partea fracţionară a nr. real x).

prof. Daniel Stretcu – Colegiul Naţional “Gh. Ţiţeica”

Page 43: RMM 8.pdf

PROBLEME PROPUSE

- 40 -

IX.10. Daca a,b ≥ 0, atunci [2a]+[2b]≥[a]+[b]+[a+b] prof. Osain Victoria

X.1. Determinaţi z,y∈Z , pentru care 2462332 +⋅=⋅ yx . Prof. Nedeianu Dan

X.2. Fie z1, z2∈C astfel încât 2008,2009 21 == zz . Să se calculeze 2

212

21 zzzz ++− Prof. Adi Lupu

X.3. Fie x . Consideram sirul

Determinati in functie de . Studiati cazul |x| Prof. Giugiuc Constantin

X.4. Sa se determine constanta a pentru care sistemul ⎩⎨⎧

−=+=++ −

axay

y

xx

3222

2

admite o unica solutie reala. Prof. Cainiceanu George

X.5. Fie 1,,2, ≠∈>∈ zCznNn , cu proprietatea ca zn=1. Sa se arate ca:

a)|1-z|>1

2−n

b) Pentru orice Zk ∈ nedivizibil cu n are loc inegalitatea |1

1|sin−

>nn

Prof. Cainiceanu George

X.6. a)Sa se arate ca functia x

xxfRRf 1)(,: −=→∗ nu este injective dar este surjectiva.

b)Se considera toate functiile injective pentru care exista m numar real nenul cu proprietatea Rxmxffxf ∈= ,))()(( o .Sa se arate ca ffff =oo si sa se deduca apoi forma generala a functiilor care verifica relatia data.

Prof. Cainiceanu George X.7. a) Să se demonstreze că dacă a şi b sunt numere supraunitare, atunci:

22

log2

log ≥+

++ baba

ba .

b) Dacă x, y sunt numere reale strict pozitive, atunci:

22

)(log2

)(log22

≥+

++

++ xyyx

xyyx

yyx

xyx

Prof. Grecu Vasile – Colegiul Economic, Turnu Severin X.8. Să se rezolve în mulţimea numerelor reale sistemul de ecuaţii:

⎪⎩

⎪⎨

=++=++=++

10log18lglglg10log12lglglg10log6lglglg

z

y

x

zyxzyxzyx

Prof. Grecu Vasile – Colegiul Economic, Turnu Severin X.9. Rezolvaţi in C ecuaţia z 3 +3z 2 -3z+1=0.

Prof. Nedeianu Dan

X.10. Dacă *,2

)Re(, +∈>∈ RkkzCz , arătaţi că kz11

− < k1 .

prof. Daniel Stretcu – Colegiul Naţional “Gh. Ţiţeica”

Page 44: RMM 8.pdf

PROBLEME PROPUSE

H SSM

- 41 -

X.11. Să se rezolve ecuaţia z3 = 5z + 47i , z = a + bi , cu a, b ∈ Z şi i2 = -1. prof. Daniel Stretcu – Colegiul Naţional “Gh. Ţiţeica”

X.12. Ai un cerc cu n puncte pe el. Fiecare 2 puncte distincte se unesc printr-un segment. Se presupune ca aceste n puncte sunt distribuite pe cerc astfel incat nu exista 3 segmente care sa se intersecteze in acelasi punct, in interiorul cercului (cu alte cuvinte, numarul punctelor de intersectie intre aceste segmente, in interiorul cercului, este maxim posibil). In cate sectiuni disjuncte este impartit interiorul cercului de catre aceste segmente?

prof. Irina Zaman – Colegiul Naţional “Gh. Ţiţeica”

XI.1. Calculaţi valoarea următoarei limite nabn

ab

n a

ab

L

ababab43421

oride

...

lim∞→

= , unde indicii reprezintă

baze de numeraţie cu }9...,,2,1{∈a si }9...,,2,1,0{∈b . Prof. Chirfot Carmen-Victoriţa

XI.2. Dacă A∈M 3 (R) cu A2 = -2A , să se arate că det (A+2I3)≥det A. Prof. Nedeianu Dan

XI.3. Calculaţi 25

]ln2[lim2

+++

∞→ xxxx

x, unde ][x partea întreagă a numărului real x.

Prof.Daniel Sitaru

XI.4. Fie ⎟⎟⎟

⎜⎜⎜

⎛=

300330333

A şi 51AB = . Să se calculeze suma elementelor matricii .BB T−

Prof.Daniel Sitaru XI.5. Fie şirurile 11 )(,)( ≥≥ nnnn yx , de numere raţionale care verifică relaţia:

2)23( nnn yx +=+ pentru orice 1≥n . Se cere:

n

n

n yx

∞→lim .

Prof.Daniel Sitaru

XI.6. Să se calculeze: x

xx

xx

xx

xx

x ⎟⎟⎠

⎞⎜⎜⎝

⎛+++

−−−−

∞→

4321

4444lim

Prof.Daniel Sitaru XI.7. Fie A,B astfel incat exista Calculati

Prof.Grecu Vasile XI.8. Fie A, B . Aratati ca daca

det|A+KB| este inversabila in Prof.Trailescu Diana

XI.9. Fie Rbaf →],[: , f functie ROLLE si derivabila pe [a,b]. Aratati ca exista o

vecinatate a lui a astfel incat f este crescatoare pe si o vecinatate a lui b astfel incat f descrescatoare pe stiind ca

Prof. Giugiuc Constantin XI.10. Fie . Determinati det in functie de tr(A) si det(A).

Prof.Giugiuc Leonard

Page 45: RMM 8.pdf

PROBLEME PROPUSE

- 42 -

XI.11. Sa se calculeze )....2793(lim 271

91

31

∞→n,daca produsul contine n factori.

Prof. Cainiceanu George XI.12. Considerăm cba ,, numere reale nenule , “ε” o rădăcină a ecuaţiei

012 =++ xx şi matricele ⎟⎟⎟

⎜⎜⎜

⎛=

42

2

11

111

εεεεE ,

⎟⎟⎟

⎜⎜⎜

⎛=

bacacbcba

F .

a) Să se arate că are loc egalitatea 12 −−= εε . b) Să se demonstreze că avem de asemenea 13 =ε . c) Să se demonstreze că 3)1)(1(det −+= εεεE . d) Calculând det F după regula triunghiului sau regula lui Sarrus să se arate că 3333det cbaabcF −−−= . e) Calculând det F folosind proprietatea de adunare a liniei trei şi a liniei doi peste linia unu, să se deducă formula

))((3 222333 cabcabcbacbaabccba −−−++++=−++ . f) Dacă cba ,, sunt cifre şi d este cel mai mare divizor comun al numerelor

cabbcaabc ,, , să se arate că numărul det F este întreg şi se divide cu d. g) Folosind formula det(E ∙F) =det E ∙ det F , să se arate că pentru

2)( cxbxaxf ++= este adevărată egalitatea )()()1(det 2εε fffF −= . Prof. Cainiceanu George

XI.13. Dacă na este un şir de numere reale pozitive cu propietatea că

43113 <+ +n

na

a, să se arate că şirul na este convergent.

Prof. Nedeianu Dan

XII.1. Se consideră inelul (A, +, • ) inel cu cel puţin două elemente , cu propietatea că

xy+yz+zx=xyz, pentru orice x,y,z ∈A-{0}. Să se arate că inelul este corp comutativ. Prof. Nedeianu Dan

XII.2. Să se calculeze ∫ ⎟⎠⎞

⎜⎝⎛∈

++

=2

,0;cos3sin2cos2sin3 πxdx

xxxxI

( ) Rxdxex

xexeeJ x

xxx

∈++

+++= ∫ ;

13256

.

Prof.Daniel Sitaru XII.3. Se defineşte pe mulţimea R legea de compoziţie x*y=10xy+10x+10y+9.

a) Aflaţi a,b∈Q–Z încât a*b∈Z b) Aflaţi a,b∈Q–Z încât a*b∈N

c) Aflaţi a,b∈R–Q încât a*b∈Z d) Aflaţi a,b∈R–Q încât a*b∈N.

Prof.Daniel Sitaru

XII.4. Dacă f : R*→ R; ( ) Rxxx

fxf ∈=⎟⎠⎞

⎜⎝⎛+ ;43 2 * să se calculeze: ( )∫

3

1

dxxf .

Prof.Daniel Sitaru

Page 46: RMM 8.pdf

PROBLEME PROPUSE

H SSM

- 43 -

XII.5. Fie ∫=2

0

cos

π

xdxxa nn . Să se calculeze: nn

annn )1(lim 22 +−+∞→

.

Prof.Daniel Sitaru

XII.6. Să se arate că matricea A=⎟⎟⎟

⎜⎜⎜

300730083006

100810081007

200620072008

200720092008200920082007200820072009

este inversabilă.

Prof.Adi Lupu

XII.7. Calculand , determinati suma .

Prof.Giugiuc Leonard XII.8. În mulţimea )( 72 ZM se consideră submulţimea M a tuturor matricilor

⎟⎟⎠

⎞⎜⎜⎝

⎛=

abbabaMˆˆˆˆ

)ˆ,ˆ( , 7ˆ,ˆ Z∈ba si matricile ⎟

⎟⎠

⎞⎜⎜⎝

⎛=

1̂0̂0̂1̂

2I , ⎟⎟⎠

⎞⎜⎜⎝

⎛=

0̂0̂0̂0̂

2O .

a) Să se verifice că MI ∈2 şi MO ∈2 . b) Să se arate că }6̂,1̂,0̂{}ˆˆ{ 7

3 =∈Zaa . c) Să se verifice că MBAMBA ∈∈⋅ ,, . d) Să se arate că ),,( ⋅+M este inel.

e) Să se arate că funcţia 7: Z→Mf , babaMf ˆˆ))ˆ,ˆ(( += este morfism de inele. f) Să se arate că ecuaţia 2007333 =++ zyx nu are soluţii numere întregi. g) Să se rezolve ecuaţia MXOX ∈= ,2

6 . Prof. Manuela Prajea

XII.9. Se consideră 0, >ba şi funcţia )1)(()(,),0(: bx

axxff ++=→∞ R .

a) Să se verifice că 1)( +++= abxabxxf .

b) Să se calculeze )(lim xfx ∞→

.

c) Să se determine ecuaţia asimptotei oblice spre ∞+ a funcţiei f . d) Să se calculeze )(xf ′ . e) Să se arate că f este convexă. f) Să se arate că, dacă R→],[: dcF este funcţie convexă atunci maximul funcţiei F este egal cu )(cF sau )(dF . g) Să se deduca inegalităţile:

89)1...11)(...(

2

2121

2 nxxx

xxxnn

n ≤++++++≤ , *N∈n , ]2,1[,...,, 21 ∈ nxxx

Prof. Manuela Prajea XII.10. Se consideră funcţiile RR →:f , crescătoare pe ),0( ∞ ,care au proprietatea:

( ))()(2)()( yfxfyxfyxf +=−++ , R∈ yx, si 1)1( =f . b) Să se verifice că funcţia RR →:g , 2)( xxg = satisface condiţiile din enunţ. c) Să se arate că: )()( xfxf =− , R∈x . d) Să se arate că )()( 2 xfrrxf = , R∈x , Q∈x . e) Să se dea exemplu de funcţie CC →:h care are proprietatea că

Page 47: RMM 8.pdf

PROBLEME PROPUSE

- 44 -

( ))()(2)()( 212121 zhzhzzhzzh +=−++ , C∈ 21 , zz . Prof. Manuela Prajea

XII.11. Se consideră funcţiile f ,g :[0 , ∝ )→R, x

xxxf+

−+=1

)1ln()( şi

xxxg −+= )1ln()( . a) Să se calculeze )(' xf şi )(' xg . b) Să se arate că 0)( ≥xf , pentru orice x ≥0. c) Să se arate că 0)( ≤xg , pentru orice x ≥ 0.

d) Să se determine valoarea integralei definite ∫1

0

)( dxxf .

e) Să se demonstreze că 2114ln2ln1 ≤−≤− .

f) Considerăm funcţia R→+= ]1,0[:),1ln()( hxxh . Să se arate că dacă notăm

∑=

=n

kn n

khs1

)( ,atunci avem ∏=

+=

n

kn n

kns1

ln .

g) Considerând cunoscut rezultatul că ∫→1

0

)(1 dxxhsn n ,să se calculeze limita şirului (xn)

dat de formula xn = n

nnnnn ))...(2)(1( +++.

Prof. Cainiceanu George

XII.12. Calculati [ ]dxxx∫ +π

0

cossin , ( notam [a]- partea întreagă a nr. real a).

Prof. Grecu Vasile – Colegiul Economic, Turnu Severin

XII.13. Calculati: dxxxxa

a∫−

−+ arcsin)1ln( 2 , unde )1,0(∈a .

Prof. Grecu Vasile – Colegiul Economic, Turnu Severin

XII.14. Calculaţi primitivele funcţiei f : (1,∞ )→R, f(x)=1

12235

24

++−−+

xxxxx

Prof. Nedeianu Dan

Page 48: RMM 8.pdf

PROBLEME PROPUSE

H SSM

- 45 -

OLIMPIADA DE MATEMATICĂ ETAPA LOCALĂ - 27 IANUARIE 2008

prof. Nanuti Dan prof. Antonie Rodica

Clasa a V-a I. Calculaţi: a) 4003291472914 22 −⋅−⋅ ;

b) ( )[ ] ( )11951752106201107 523:552:233 −+⋅−+⋅ ; c) 2008200720062005...987654321 +−+++−++−++−+ ;

Mariana Tatucu II. În două lăzi se află un număr diferit de mere . Dacă se mută din lada mai plină în lada mai goală atâtea

mere câte sunt în lada goală, după trei operaţiuni de acest fel, în fiecare ladă rămân câte 240 de mere.

Câte mere au fost la început în fiecare ladă? Rodica Antonie

III. Să se compare numerele Acarda = şi Bcardb = , unde { }20082007 22, ≤∈= xINxxA p şi { }133831, ≤≤∈= yINyyB .

Valeria Baloi IV. Să se determine INba ∈, astfel încât 137172326 −⋅=+⋅+ + babaa .

Manuela Prajea Clasa a-VI-a

I. Să se determine suma tuturor numerelor de forma abc , ştiind că cabcab ,, sunt direct proporţionale

cu accbba +++ ,, . II. Aflati cel mai mic număr natural care se scrie, în baza 10, numai cu cifrele 2 şi 3 şi se divide cu 132. III. a)Se dau punctele CBA ,, şi D , în această ordine, astfel încât ACADAB ⋅=⋅+⋅ 954 şi .18=BD

Să se afle lungimea segmentelor BC şi CD . b)Fie semidreptele [[[ OCOBOA ,, şi [OD , în aceasta ordine, astfel încât ( ) ( )BOCmAODm ∠⋅=∠ 2 şi ( ) oXOYm 90=∠ , unde [OX şi [OY sunt bisectoarele unghiurilor AOB∠ şi COD∠ . Să se

determine măsura unghiului BOC . IV. Se consideră egalitatea : yxyx 344 227 ⋅+⋅= , ., INyx ∈ a)Arătaţi că perechile 3=x , 1=y şi respectiv 3,9 == yx verifică egalitatea de mai sus. b)Arătaţi că egalitatea din enunţ este verificată de o infinitate de perechi de numere naturale x şi y .

Dan Nedeianu Clasa a-VII-a

I. Dacă abc192 este număr natural, să se arate că numărul ( )( )cba 21 ++ este pătrat perfect. II. a)Câte soluţii întregi are inecuaţia : .2008≤x

b) Câte soluţii naturale are ecuaţia: .2008=++ zyx III. Se consideră trapezul isoscel ABCD cu baza mare AB , ( ) 060=∠ABDm , { }OBDAC =∩ , E mijlocul diagonalei [ ]AC şi F mijlocul diagonalei [ ]BD . Dacă M este punctul egal depărtat de vârfurile CBA ,, şi D ale trapezului, iar G este centrul de greutate al triunghiului OEF , să se arate că G este mijlocul segmentului [ ]MO .

Dan Nedeianu IV. Se consideră dreptunghiul ABCD cu aAB = , bAD = şi a >b . Bisectoarea unghiului BAD intersectează BD în E şi BC în F , iar paralela prin E la AB intersectează AC în G . a)Determinaţi lungimea segmentului [ ]EG . b)Demonstraţi că FG ⊥ .BD

Selectate de Dana Paponiu si Carmen Coada

Page 49: RMM 8.pdf

PROBLEME PROPUSE

- 46 -

Clasa a-VIII-a

I. Dacă a >b >0 şi abba 1022 =+ , calculaţi:baba

−+ şi

baba

+− .

Octavian Ungureanu II. Dacă 2, ≥∈ nINn şi x >0, să se arate că ( )( ) ( ) 0221...1 2 ≥+−+++++ nnn xnxxxx

Dan Nedeianu III. Fie ABCD tetraedru cu propietăţile ( ) ( ) ( ) ( ) ( )BDADBCACAB ≡≡≡≡ şi AD ⊥ BC .

Să se demonstreze că ABCD este tetraedru regulat. Iuliana Gimoiu

IV. Se consideră triunghiul ABC dreptunghic cu ( ) oAm 90=∠ , cmACcmAB 8,6 == , iar punctele D şi E sunt mijloacele segmentelor ( )AB , respectiv ( )AC . Pe prelungirea segmentului ( )DE se iau punctele M şi N astfel încât triunghiurile MAB şi NAC să nu fie obtuzunghice , iar

.12cmMN = În punctul M se ridică perpendiculara VM ⊥ ( )ABC , cmVM 1= . a)Să se demonstreze că ( ) oANCm 90=∠ . b)Să se calculeze distanţa de la punctul V la dreapta AB .

Dan Nedeianu Clasa a IX-a

I. a.Fie ( ){ }5, ≤+= yxyxA . Să se reprezinte grafic mulţimea A folosind ca unitate de măsură “1cm”. b.Să se arate că oricum am lua 101 puncte distincte din mulţimea A, există cel puţin două dintre

acestea aflate la o distanţă mai mică sau egală cu “1cm” unul de altul. Daniel Sitaru

II. Fie Rcba ∈,, variabile cu proprietatea că 18=++ cba . Să se determine valoarea maximă a

produsului 432 cba şi să se determine cba ,, în acest caz. George Cainiceanu

III. Să se rezolve ecuaţia

⎭⎬⎫

⎩⎨⎧ +

=⎭⎬⎫

⎩⎨⎧ −

31

21 xx

, în mulţimea numerelor reale, unde { }x este partea

fracţionară a numărului real x . Daniel Sitaru

IV. Pe laturile [ ] [ ]ACAB , ale ABC∆ se consideră punctele ,D respectiv ,E astfel încât

0=+++ ECEADBDA . Dacă O este mijlocul segmentului [ ]DE şi { }FBCAO =I să se

calculeze raportul FCBF .

Dan Nedeianu Clasa a X-a

I. Să se rezolve ecuaţia: 2

1 327

181

32727 +

−+=++ x

xxx .

Dan Nedeianu II. a)Fie Rzyx ∈,, . Să se demonstreze inegalităţile:

222222444 xzzyyxzyx ++≥++ , ( )zyxxyzzyx ++≥++ 444 . b)Să se demonstreze că 27log25log7log7log5log3log 222

42

42

42 ⋅⋅>++ .

Daniel Sitaru

III. Fie Cz ∈ astfel încât 1≤z . Să se arate că ( ) 20092008

1

211 <⋅++ ∑=k

kk zi .

Ovidiu Ticusi IV. Se consideră ecuaţia (E) .01828 =−− xx

Page 50: RMM 8.pdf

PROBLEME PROPUSE

H SSM

- 47 -

a)Dacă z este o rădăcină de modul 1 a ecuaţiei (E), să se arate că 1120 =−z . b)Să se determine toate rădăcinile complexe de modul 1 ale ecuaţiei (E).

George Cainiceanu Clasa a XI-a

I. Se dă matricea: ⎟⎟⎠

⎞⎜⎜⎝

⎛−−

=2008200720092008

A .

a)Să se arate că ( ) ( ) 222 det OIAATrAA =⋅+⋅− , unde TrA este suma elementelor de pe

diagonala principală a matricei A. b)Să se calculeze

2008A şi 2009A . Adi Lupu

II. Se dă şirul de numere reale ( ) Nnnx ∈ pentru care ,200820071 +=+ nn xx 10 −=∈ xsiNn . a)Să se determine termenul general al şirului. b)Să se studieze monotonia, mărginirea şi existenţa limitei şirului.

Doru Presneanu III. Fie ( )RMA 3∈ . Ştiind că ,AAt α= unde { }1−∈ Rα , să se calculeze

2008det A . Adi Lupu, Doru Presneanu

IV. Calculaţi ( )3 23 563coslim +++∞→

nnnnn

π . Dan Nedeianu

Clasa a-XII-a I. Pe IR se consideră legea : 10663 +++=∗ yxxyyx . a)Să se arate că : ( )( ) 2223 −++=∗ yxyx ; b)Să se rezolve ecuaţia : 23... 4015

2008

−=∗∗∗∗ 44 344 21ori

xxxx .

Emilia Raducan

II. Fie ⎟⎠⎞

⎜⎝⎛−=

aaG 1,1

1 şi ⎟⎠⎞

⎜⎝⎛−=

bbG 1,1

2 , unde a>0 şi b>0 şi ba ≠ .

Definim legile : 12 ,,1

Gyxxyayxyx ∈

++

=∗ şi 22 ,,1

Gyxxybyxyx ∈

++

=o .

a)Arătaţi că ( )∗,1G şi ( )o,2G sunt grupuri abeliene. b)Arătaţi că ( )∗,1G ≈ ( )o,2G .

Daniel Sitaru

III. Fie ( ) dxxxxF ∫= 2008sin

2006cos2007 , unde ⎥⎦⎤

⎢⎣⎡∈

2,

4ππx . Ştiind că 0

4=⎟

⎠⎞

⎜⎝⎛πF să se afle ⎟

⎠⎞

⎜⎝⎛

2πF .

Eleodor Popescu

IV. a)Aratati că există IRdcba ∈,,, cu ( ) ( )( ) ⎥⎦⎤

⎢⎣⎡∈++=++

2,0,cossincossin1 2 πxxdcxbaxx .

b)Să se calculeze:

( )( )dx

xxxx

∫ ++++

cos1sin1cossin1 , unde ⎥⎦

⎤⎢⎣⎡∈

2,0 πx .

Dan Nedeianu NOTA DIN PARTEA REDACTIEI Ne cerem anticipat scuze colegilor care sunt autori ale unor probleme din concurs si nu sunt trecuti ca autori. Le solicitam totodata sa ne anunte pentru a prezenta cuvenita erata.

Page 51: RMM 8.pdf

PROBLEME PROPUSE

- 48 -

Test de verificare – admitere clasa a V-a

Colegiul National „Gheorghe Ţiţeica” MATEMATICĂ

SUBIECTUL I Aflaţi termenul necunoscut din: [ 35 – 4 × ( 24 : 3 ) – ( a + 13 ) : 10 ] ×2 + 12 : 3 = 6 SUBIECTUL II 1) Suma dintre câtul şi restul unei împărţiri de numere naturale nenule este 30 . Restul este cu 2 mai mare

decât triplul câtului. Determinaţi deîmpărţitul ştiind că suma dintre acesta şi împărţitor este 2023. 2) Într-o urnă sunt 105 bile albe, negre şi roşii. Dacă extragem din urnă 81 de bile cel puţin una este albă,

dacă extragem 61 de bile cel puţin una este roşie şi dacă extragem 71 de bile cel puţin una este neagră.

Câte bile sunt de fiecare culoare?

LIMBA ROMÂNĂ I Se dă textul: „ Fantastic joacă rândunici zglobii În cerul plin de umbră şi lumină, Şi-i linişte adâncă în grădină, Sub piersicii cu flori trandafirii …” (Şt. O. Iosif – Linişte ) Cerinţe:

1) Găsiţi câte un cuvânt cu înţeles asemănător pentru următorii termeni din text: fantastic, zglobii, linişte, adâncă , trandafirii.

2) Despărţiţi în silabe cuvintele: zglobii, flori, rândunici, piersicii, umbră. 3) Transcrieţi din text două cuvinte cu înţeles opus şi precizaţi ce sunt ca părţi de vorbire 4) Precizaţi funcţia sintactică (parte de propoziţie) a cuvintelor: (în) cerul , -i , (cu ) flori, rândunici,

trandafirii. 5) Alcătuiţi o propoziţie după următoarea schemă:

S At P C C subst. subst. vb. altă parte de vorbire subst. II Realizaţi o compunere de 15 – 20 de rânduri în care personajul principal să fie o rândunică. Daţi un titlu

potrivit compunerii. NOTĂ: Timp de lucru 2 ore. Toate subiectele sunt obligatorii. Se acordă 10p din oficiu.

Colegiul National „Gheorghe Ţiţeica” MATEMATICĂ

SUBIECTUL I Aflaţi termenul necunoscut din: { [ 5 × ( a + 21 ) – 55 × 4]:5 + 27:3}:10=4 SUBIECTUL II 1) Trei copii au împreună 2800 lei. Să se afle cât are fiecare copil ştiind că primul are cât ceilalţi doi

împreună, iar al doilea cu 100 lei mai mult decât al treilea. 2) Într-o urnă sunt 105 bile albe, negre şi roşii. Dacă extragem din urnă 81 de bile cel puţin una este albă,

dacă extragem 61 de bile cel puţin una este roşie şi dacă extragem 71 de bile cel puţin una este neagră.

Câte bile sunt de fiecare culoare?

Page 52: RMM 8.pdf

PROBLEME PROPUSE

H SSM

- 49 -

LIMBA ROMÂNĂ I Se dă textul: „Sub streşini cuiburile vechi Ascund iar vesele perechi De rândunele, Şi cuiburi nouă se clădesc, Iar eu, furat de-un vis, privesc Zâmbind la ele …” (Şt. O. Iosif – Cântec) Cerinţe:

6) Găsiţi câte un cuvânt cu înţeles asemănător pentru următorii termeni din text: vechi, (ascund) iar, se clădesc, vesele , zâmbind.

7) Despărţiţi în silabe cuvintele: streşini, vechi, rândunele, cuiburile, perechi. 8) Precizaţi ce sunt ca parte de vorbire următoarele cuvinte din text: vechi , ascund , nouă, zâmbind,

ele. 9) Realizaţi schema completă a următoarei propoziţii (parte de propoziţie + parte de vorbire) :

„Sub streşini cuiburile vechi Ascund iar vesele perechi De rândunele …”

II Realizaţi o compunere de 15 – 20 de rânduri în care să descrieţi sosirea anotimpului preferat. Daţi un

titlu potrivit compunerii. NOTĂ: Timp de lucru 2 ore. Toate subiectele sunt obligatorii. Se acordă 10p din oficiu.

Colegiul National „Traian”

14.06.2008 MATEMATICA

1) a) Efectuati =−−×+ 5:]132:)2:81742[( 2p b) Determinati numarul a din egalitatea 526:648)2403(:480 =−−× a 2p

2) Daca adunam triplul unui numar cu triplul altui numar , obtinem 150. Aflati numerele stiind ca dublul diferentei lor este 40. 3p 3) a).Scrieti toate numerele naturale de doua cifre diferite care au suma cifrelor egala cu 4. 1p b).Scrieti toate numerele naturale de cel mult trei cifre care au suma cifrelor egala cu 4. 1p

LIMBA SI LITERATURA ROMANA Citeşte cu atenţie textul următor, pentru a răspunde cerinţelor:

Azi e prima zi de şcoală. Trei luni de vacanţă la ţară s-au spulberat ca un vis. De dimineaţă, mama

m-a dus la şcoală, ca să mă înscrie într-a treia primară; nu eram prea încântat să mă duc, căci gândul

zbura încă la ţară. Pe toate străzile, forfotă mare de copii; în cele două librării, o mulţime de taţi şi mame

se îngrămădeau să cumpere ghiozdane cu cureluşe, serviete şi caiete, iar în faţa şcolii se înghesuia atâta lume! Omul de serviciu şi gardianul se străduiau din răsputeri să elibereze poarta. ( Edmondo de Amicis, Cuore, inimă de copil) 1. Când se petrece acţiunea din fragmentul citat? 0,5p 2. Unde şi-a petrecut copilul vacanţa de vară? 0,5p 3. Din ce cauză copilul nu era prea încântat că trebuie să meargă la şcoală? 0,5p 4. Găseşte cuvântul cu acelaşi înţeles, apoi cuvântul cu înţeles opus pentru: prima şi să cumpere. 1p 5. Alcătuieşte schema următoarei propoziţii: Trei luni de vacanţă la ţară s-au spulberat ca un vis 1,5p 6. Alcătuieşte două propoziţii în care substantivul vis să aibă funcţiile sintactice de subiect, respectiv, de

atribut. 1p 7. Imaginează-ţi că eşti personajul din textul dat. Continuă şirul întâmplărilor, folosind ca început

enunţul: Omul de serviciu şi gardianul se străduiau din răsputeri să elibereze poarta. Compunerea ta să aibă 12 - 15 rânduri. Găseşte-i un titlu potrivit . 4p

Page 53: RMM 8.pdf

PROBLEME PROPUSE

- 50 -

Colegiul National „Traian” 20.06.2002

MATEMATICA 1) Determina valoarea lui x daca 6}8:]131)342(53[4{7303 =×+−×××+−×+× x . 2) Daca adunam triplul unui numar cu triplul altui numar obtinem 165.Aflati numerele stiind ca triplul

diferentei numerelor este 45. 3) Suma a trei numere naturale este 6400. Daca se imparte primul numar la cel de-al doilea se obtine

catul 2 si restul egal cu cel de-al treilea numar. Aflati numerele stiind ca al treilea este mai mic cu 100 decat al doilea.

LIMBA SI LITERATURA ROMANA

1) Transforma vorbirea directa in vorbire indirecta: “-Buna seara mos Martine! -Buna vreme ,mai copile! -Incotro cu noaptea-n cap? -La oras sa-mi iau un brad. -Pai in codru ,brain u ai? -am,da-mi pare rau sa-I tai!”

2) Ilustreaza in enunturi proprii scrierea cuvintelor: Nu mai | numai | nu m-ai; cai | ca-I; mie | mi-e; ai | a-i.

3) Analizeaza grammatical (sintactic si morfologic) cuvintele din versurile: “Voi ati vantat cu glas fierbinte Naturii calde imnuri sfinte.” (George Cosbuc—Vestitorii primaverii)

Colegiul National „Traian” 20.06.2001

MATEMATICA 1) Afla valoarea lui a din egalitatea 100081253223:]40057)4:3:33920[( =×+×−×−×++ a 2) Suma a doua numere impartita la diferenta lor este 10 rest 1. Care sunt numerele, stiind ca unul este

mai mare decat celalalt cu 3. 3) Doi colegi citesc aceeasi carte. Dupa ce primul a citit 165 de pagini, iar cel de-al doilea 225 de pagini,

se constata ca primul mai are de citit de cinci ori mai mult decat al doilea. Cate pagini are cartea? LIMBA SI LITERATURA ROMANA

1) “Un satean avea trei feciori care se tot certa. Din pricina aceasta el era tare suparat. Intr-o zi el ii chema pe toti si le zise:

-care dintre voi ar putea sa franga acest manunchi de nuiele, capata o mie de lei. Incercara pe rand ,dar n- au reusit. -E peste putinta,zisera ei. -Ba sa vedeti ca se poate. Batranul dezlega manunchiul si franse nuielele una cate una. -Apoi asa stim si noi,zisera feciorii. -Vedeti, asa e si cu voi. Daca veti fi uniti, veti fi puternici, daca nu, va poate frange oricine. Printre feciori se lasa o tacere de mormant” (“Tatal si feciorii”)

a) Povestiti textul, transformand vorbirea directa in vorbire indirecta. b) Gasiti sinonime pentru cuvintele “manunchi”, “feciori”, “din pricina”, “sa franga”; alcatuiti cu

fiecare cate o propozitie.

c) Alcatuiti o propozitie dupa schema ... psvadjs

ASPAC.

Page 54: RMM 8.pdf

PROBLEME PROPUSE

H SSM

- 51 -

Colegiul National „Traian” 16.06.2007

MATEMATICA 1) a)Sa se efectueze : .4]26:)1519491[( ×−+×−

b)Sa se afle cate numere naturale de trei cifre au cifra sutelor egala cu dublul cifrei unitatilor, iar cifra zecilor este mai mica decat cifra unitatilor.

2) Un sfert din lungimea unui teren dreptunghiular este cu 2m mai mare decat a treia parte din latime. Sa se afle dimensiunile dreptunghiului stiind ca perimetrul sau este de 72m.

LIMBA SI LITERATURA ROMANA

1) Alcatuiti enunturi cu urmatoarele ortograme: va|v-a; mai|m-ai; car|c-ar; iau|i-au. 2) Se da textul:

Soarele scapata spre asfintit, departe, printer trestii. Inainte de fiorul amurgului,balta tacea, solemna, oglindind in limpezimile ei un cer inalt, fara pata.

a) Indicati cuvinte cu acelasi inteles pentru: a scapata, asfintit, cer, a oglindi. b) Analizati cuvintele subliniate din textul de mai sus.

3) Redactati o compunere de 15-20 de randuri, in care sa imbinati naratiunea si dialogul si care sa aiba urmatorul sfarsit: …De-atunci nu m-am maid us niciodata cu tema nefacuta. Gasiti-I un titlu potrivit

Page 55: RMM 8.pdf

Concursuri

- 52 -

OLIMPIADA NATIONALA DE MATEMATICA 2007 - 2008

Prencea Cassian V Medalia de bronz Paponiu Dana Puican Tiberiu VI Medalie de aur,Mentiune MEDC Prajea Manuela Stefan Andrei VII Medalie de bronz Prajea Manuela Nistor Andreea VIII Medalie de bronz Ionica C-Tin Carapencea Constantin X Medalie de bronz,Mentiune MEDC Paponiu Dana Tigora Andrei XII Med.Bronz Cainiceanu Gh.

Olimpiada de matematicã - etapa judeţeană 01.03.2008 - Clasa a V-a

Prencea Cassian CNT I Barzuica Alexandru Sc.nr.3 M Verghelet Maria CNT II Botezatu Diana CNT M Nicolicioiu Armand Titeica III Glonti Madalina CNT M Pogacean Victor CNT III Grecu Andreea H CNT M Nitoiu Mihai CNT M Rosoga Augusta Lic.Pedagogic M Dirpes Iulian Menandu Sc.Voinescu M Amzoi Ileana CNT M Cebuc Razvan CNT M Arnautu Catalina Sc.14 M Prundeanu Sabina Sc.14 M Boruga Bogdan CNT M Banica Tudor CNT M Chesa Maria Severinesti M Chirita Dan CNT M Duican Irina Stefania Sc. nr. 14 M Mihutescu Diana Sc.M.Viteazul M Dunarintu George Titeica M Radoslav Teodor CNT M Lascu Robert CNT M Ciuciu Simona CNT M Ochea Alin Titeica M Daescu Maria Lic.Pedagogic M Pirvulescu Bianca Sc.P.Segescu M Durus Andrei Sc.Voinescu M Robu Silviu Mihai Sc.14 M Plavitu Victor CNT M Sacarin Claudiu CNT M Stefanoiu Cristian CNT M Suta Alexandru CNT M Andreescu Roxana CNT M Vaduva Beatrice G CNT M Atomei Ciprian CNT M

Clasa a VI-a Ionas Ioana Madalina Titeica I Baloi Elisabeta Sc. 2 M Puican Tiberiu CNT II Popescu Corneliu CNT M Ciuciu Laurentiu CNT III Iosu Raluca Sc. nr 6 M Spinu Antonela Cristina Sc.Sergescu III Vladu Catalin Sc.Negreanu M Ciobanu Diana CNT M Berbecariu Elena Sc.14 M Ciuta Cora CNT M Borozia Daniel Cosmin Sc.Sergescu M Troncota Diana CNT M Bosoanca Adriana D Sc.14 M Benga Andrei CNT M Diaconescu Ovidiu G Sc.6 M Gherasim Andrei CNT M Dragomir Andrei CNT M Soare Daniela Ioana Sc.14 M Ene Bogdan Titeica M Badescu Codrin CNT M Lupuleti Catalin Titeica M Barbulescu Ramona CNT M Marghescu Luminita M Titeica M Barbulescu Ramona Sc.14 M Petrescu Rucsandra V Sc.Sergescu M Bojinovici Alexandra CNT M Prencea Vlad CNT M Gheorghe Bogdan CNT M Semen Alexandra CNT M Marzoca Cezara CNT M Visan Cosmina CNT M Yassin Aylin CNT M

Page 56: RMM 8.pdf

Concursuri

- 53 -

H SSM

Clasa a VII-a Numele şi prenumele Scoala Pr. Numele şi prenumele Scoala Pr.

Ştefan Andrei CNT I Driga Darie CNT M Georgescu Ana CNT II Gavriloiu Costin Ţiţeica M Tănasie Denisa CNT III Păcuraru Adrian Şc.Orşova M Nicolae Andrei CNT M Păvălaşe Bogdan Ţiţeica M Nuţă Flavius CNT M Băcanu George Şc. 6 M Zamfirescu Simona CNT M Ciontea Ştefan Ţiţeica M Chiriş Alexandra CNT M Ciucur Ştefan Ţiţeica M Marghescu Andreea CNT M Gimoiu Ruxandra CNT M Sbîrcea Alexandra CNT M Mitran Dragoş CNT M Ţolea Cristian Ţiţeica M Voicu Răzvan CNT M Boboiceanu Daniel Şc.14 M Arbănaşi Emil Ţiţeica M Nistor Andreea Ţiţeica M Badea Beatrice CNT M Săceanu Andrei CNT M Cătănescu Raluca Şc.Voinescu M Ţirlui Valeria Ţiţeica M Diaconu Paula Lic.Odobleja M Florescu Alexandru Ţiţeica M Dincă Diana Col.Decebal M Anghel Cosmin Lic.V.Gomoiu M Gherghe Ana Maria Şc.Voinescu M Anghel Cristian Lic.V.Gomoiu M Mituleţu Mădălina Ţiţeica M Băcanu Alexandru CNT M Pristoleanu Narcis CNT M Vâlcu Andrei CNT M Tripcea Roxana Şc.14 M Aniţa Georgiana CNT M Zorilă Mihai CNT M Borcean Diana Mădălina Şc. Severinesti M

Clasa a VIII-a Numele şi prenumele Scoala Pr. Numele şi prenumele Scoala Pr.

Nistor Adriana Şc.14 I Cîrciu Bogdan Ţiţeica M Nicolicioiu Andrei Ţiţeica II Constantinescu Robert Şc.Voinescu M Prunescu Flavius CNT II Filipescu Alexandru Şc.14 M Andreescu Mădălina CNT III Florichescu Alexandru Şc.Orşova M Asproniu Robert CNT M Frăţilă Remus Şc.Voinescu M Buţă Alexandra Simona Şc.Strehaia M Pănescu Dragoş CNT M Toader Simona Şc.Voinescu M Vlaston Cristina Şc.6 M Pacioagă Florentina Şc. Sergescu” M Zugravu Rozalia CNT M

Clasa a IX-a Clasa a X-a Numele şi prenumele Scoala Pr. Numele şi prenumele Scoala Pr.

Teşilă Bianca CNT I Şeitan Mihaela CNT I Papa Florin CNT II Carapencea Constantin CNT II Popa Bogdan CNT III Pîrvulescu Dan CNT III Văcărescu Mirela A Ţiţeica M Piţ Rada Andrei CNT M Arcuşi Anamaria CNT M Calinovici Paul CNT M Bălulescu Ligia CNT M Iorga Cristian Ţiţeica M Radu Cornel Ţiţeica M Alexandru Crivac Cristina CNT M Ştefan Radu Ţiţeica M Croitoru Răzvan CNT M Copăceanu Adela Ţiţeica M Duţă Adrian CNT M Curea Guţă Marius Ţiţeica M Marinescu Mihai CNT M Doroiman Andreea Ţiţeica M Damian Beatrice CNT M Dulan Ana CNT M Lolea Iulian CNT M Fulga Iulia CNT M Ciotîrlă Dănuţ CNT M Furcuţă Ioana CNT M Crăciunescu Marian CNT M Glisca Marian CNT M Nicoară Călin CNT M Milici Alina CNT M Petrescu Bianca CNT M Pană Florin Claudiu CNT M Băltărete Ileana CNT M Papa Mădălina CNT M Ciuciu Lucian Florin Ţiţeica M Crîng Alisa Pedagogic M

Page 57: RMM 8.pdf

Concursuri

- 54 -

Clasa a XI-a Clasa a XII-a Numele şi prenumele Scoala Pr. Numele şi prenumele Scoala Pr.

Puţan Cătălin Ţiţeica I Tigora Andrei CNT I Budirinca Alexandru CNT II Coandă Oana CNT II Ciorobea Mihai CNT II Gogoloiu Gabriela Ţiţeica III Dumbravă Roxana T. Lalescu III Mitucă Atena CNT M Camalesa Bogdan Ţiţeica M Prundeanu Andreea CNT M Dunăreanu Lidia Ţiţeica M Hinoveanu Cătălin CNT M Mercioni Marina T. Lalescu M Popescu Anca Pedagogic M Purcaru Alin Ţiţeica M Tuţă Leontin CNT M Răducu Alexandru CNT M Răveanu Ioana CNT M Şuşelea Robert CNT M Botea Silvia CNT M Ilie Mariana Pedagogic M Lică Ştefania T. Lalescu M Porojan Otilia CNT M

CONCURSUL DE MATEMATICĂ "ADOLF HAIMOVICI"

- etapa judeţeană 01.03.2008 - Clasa a IX-a

Bran Adelina Ţiţeica I Ţîrlui Andreea T. Lalescu M Dumitraşcu Roxana Economic II Alstani Cătălin D. Tudor M Moraru Rebeca Pedagogic II Siclitaru Alexandra Economic M Sitaru Daniela T. Lalescu II Sîbu Denisa Economic M Vlădescu Mădălina Economic II Spătaru Alexandra Pedagogic M Răescu Ionela Mihaela D. Tudor III Suta Alexandra CNT M Sîrbu Denisa Ţiţeica III Ţogoe Luiza Economic M Cunete Mihaela Decebal M

Clasa a X-a Bobia Andreea Decebal I Dumitrana Manuela Economic II Colţatu Elena Decebal I Ilie Liviu Costin Auto II Avrămiuc Andrei Strehaia II Lupu Ana Strehaia II Cazacu Laurnţiu Auto II Peagu Mihai Bogdan Auto II Chiosa Răducu C-tin Auto II Şerban Bogdan D. Tudor II Chiuescu Alina CNT II

Clasa a XI-a Croitoru Bogdan Ţiţeica I Predescu George Ţiţeica M Coliţă Georgiana Economic II Andriţa Caius Economic M Anghel Nicolae Mihail Strehaia III Bădeţi Daniela Ţiţeica M Prundeanu Marioara Economic III Giurescu Ştefania Ţiţeica M Fica Andreea Ţiţeica III Iacobescu George D. Tudor M Stoican Elena Economic III Strinoiu Eugen D. Tudor M Vinjanu Ionel D. Tudor III Zamfirescu Daniel D. Tudor M Bivolariu Ana Mari Strehaia M

Clasa a XII-a Panfil Denisa Ţiţeica I Gheorghişan Paula Construcţii III Atanasoaei Andrei Ţiţeica II Marian Andrei Vasile Ţiţeica M

Concursul national “Vranceanu-Procopiu”, Bacau,2007

Tesila Bianca IX mentiune prof. Paponiu Dana prof. Carapencea Vali

Page 58: RMM 8.pdf

Concursuri

- 55 -

H SSM

BARAJUL NATIONAL CANGURUL BUCURESTI

Tigora Andrei XII CNT PR.II Carapencea Constantin X CNT PR.II Sosu Cristian X CNT PR.II Nicolicioiu Andrei VIII CNGT PR.III Grosu Vlad VIII CNGT PR.III

BARAJUL JUDETEAN CANGURUL LA CLASELE 4-6 Popescu Cristiana IV Banica Teodor V Raveanu Robert IV Cebuc Razvan V Ion Miruna IV Verghelet Maria V Benga Cristina IV Pogacean Victor V Arbanasi Eliza IV Benga Andrei VI Capastraru Bogdan IV Mema Serban VI Butaru Nadina IV Ionas Ioana VI Chicet Clara IV Puican Tiberiu VI Chirita Dan V Troncota Diana VI Prencea Cassian V Petrescu Lorena VI

Concursul National de Evaluare in Matematica

CALIFICATI SI PREMIANTII LA ETAPA NATIONALA CONFORM SITE-ULUI FUNDATIEI Tigora Andrei XII CNT premiul.II Voicu Razvan VII CNT Puican Tiberiu VI CNT premiul.III Nistor Adriana VIII CNGT Nicoara Calin X CNT premiul.III Nistor Mihaela VIII GEN.14 Nicolicioiu Armand V CNGT Radu Cornel IX CNGT Ionas Ioana VI CNGT Stefan Radu IX CNGT Lupuleti Catalin VI CNGT Carapencea C-Tin X CNT Florescu Alexandru VII CNGT Stretcu Otilia X CNGT Tanasie Denisa VII CNT Coanda Oana XII CNT Tarlui Valeria VII CNGT

Concursul interjudetean “N.Coculescu”, Slatina, 2007 proba din 30.XI.2007 proba din 1.XII.2007 Pogacean Victor V mentiune Paponiu D Prencea Cassian V mentiune Paponiu D Puican Tiberiu VI mentiune Prajea M Pogacean Victor V mentiune Paponiu D Tesila Bianca IX premiul III Paponiu D Puican Tiberiu VI mentiune Prajea M Carapencea C-tin X mentiune Paponiu D Tesila Bianca IX mentiune Paponiu D Croitoru Razvan X mentiune Paponiu D Carapencea

Constantin X mentiune Paponiu D

Concursul interjudetean de matematica “Ion Ciolac”,

Craiova, 12.IV.2008

Prencea Cassian V mentiune prof. Paponiu Dana Pogacean Victor V mentiune prof. Paponiu Dana Puican Tiberiu VI premiul III prof. Prajea Manuela Benga Andrei VI mentiune prof. Prajea Manuela Stefan Andrei VII premiul I prof. Prajea Manuela Zamfirescu Simona VII mentiune prof. Gimoiu Iuliana Tesila Bianca IX premiul I prof. Paponiu Dana Carapencea Constantin X mentiune prof. Paponiu Dana

Page 59: RMM 8.pdf

Concursuri

- 56 -

Concursul national al revistei “Arhimede” Etapa locala, 19.XI.2007, Craiova Pogacean Victor V premiul II prof. Paponiu Dana Prencea Cassian V premiul II prof. Paponiu Dana Banica Teodor V premiul III prof. Paponiu Dana Chirita Dan V premiul III prof. Paponiu Dana Suta Adrian V premiul III prof. Paponiu Dana Carapencea Constantin X mentiune prof. Paponiu Dana Etapa zonala,Craiova , 23.II.2008 Pogacean Victor V premiul I prof. Paponiu Dana Banica Teodor V premiul II prof. Paponiu Dana Prencea Cassian V premiul II prof. Paponiu Dana Carapencea Constantin X premiul II prof. Paponiu Dana Etapa nationala, Craiova, 19.IV.2008 Pogacean Victor V premiul III Banica Teodor V mentiune Prencea Cassian V mentiune Suta Adrian V mentiune Carapencea Constantin X premiul II

Concursul interjudetean de matematica “Micul Arhimede”,Craiova Pogacean Victor V mentiune prof. Paponiu Dana Puican Tiberiu VI premiul I prof. Prajea Manuela Ciuciu Laurentiu VI mentiune prof. Prajea Manuela Benga Andrei mentiune prof. Prajea Manuela

Concursul national de matematica - “La scoala cu ceas”, Rm.Vilcea

proba de matematica Verghelet Maria V premiul III si medalie de bronz prof. Paponiu Dana Pogacean Victor V mentiune prof. Paponiu Dana Prencea Cassian V mentiune prof. Paponiu Dana Puican Tiberiu VI premiul III si medalie de bronz prof. Prajea Manuela Zamfirescu Simona VII premiul III si medalie de bronz prof. Gimoiu Iuliana Badea Beatrice VII premiul III si medalie de bronz prof. Prajea Manuela Stefan Andrei VII mentiune prof. Prajea Manuela Voicu Razvan VII mentiune prof. Prajea Manuela Nicolae Andrei VII mentiune prof. Prajea Manuela Prunescu Flavius VIII mentiune prof. Cainiceanu Gh. Andreescu Madalina VIII mentiune prof. Antonie Rodica proba tip ONM pentru clasele V-VI Prencea Cassian V medalie de bronz prof. Paponiu Dana Puican Tiberiu VI medalie de bronz prof. Prajea Manuela Benga Andrei VI medalie de bronz prof. Prajea Manuela

Concursul Interjudetean Gheorghe Titeica 23.05.2008 Individual Probe pe echipaje Verghelet Maria V CNT PR.I Prunescu Flavius VIII CNT PR.I Prencea Cassian V CNT PR.II Stefan Andrei VII CNT PR.I Pogacean Victor V CNT M Nistor Andreea VIII GEN.14 PR.I Puican Tiberiu VI CNT PR.II Tesila Bianca IX CNT M Prunescu Flavius VIII CNT M Seitan Mihaela X CNT M Tesila Bianca IX CNT M Pirvulescu Dan X CNT M Carapencea Constantin X CNT M Ciorobea Mihai XI CNT PR.II Seitan Mihaela X CNT M Andreescu Madalina VIII CNT PR.III Pirvulescu Dan X CNT M Tigora Andrei XII CNT M

Page 60: RMM 8.pdf

Concursuri

- 57 -

H SSM

Concursul Interjudeţean "Petre Sergescu" - 2008

clasa a IV-a

Burtea Cătălin Şc.Voinescu I Dragotescu Radu Andrei Şc. 6 III Popescu Cristiana Şc.Paulian I Filip Radu Şc. 6 III Paleacu Cosmin Şc.Voinescu II Lică Robert Şc.Voinescu III Şontea Claudiu Şc.14 II Tănasie Dănuţ Şc.Voinescu III Butaru Nadina Mădălina Şc.14 III Bălu Smaranda Şc.Voinescu III Crăciun Madălina Şc.Voinescu III

clasa a V-a clasa a VI-a

Pogăcean Victor CNT I Puican Tiberiu CNT I Calotă Dragoş CN.Carol II Trofin Raluca CN.Carol I Chiriţa Dan CNT III Troncotă Diana CNT II Benga Andrei CNT III clasa a VII-a clasa a VIII-a

Ştefan Andrei CNT I Nistor Adriana Şc.14 I Tănasie Denisa CNT II Băleanu Andrei Motru II Anghel Cristian Lic.Gomoiu III Andreescu Mădălina CNT III Gimoiu Ruxandra CNT III Semenescu Anca CD.Loga III clasa a IX-a clasa a X-a

Teşilă Bianca CNT I Padureanu Victor CN Carol I Miroşu Raluca CNT II Pirvulescu Dan CNT II Agape Mihai CNT III Birovescu Georgiana CNT III Alexandru Bogdan CN.Carol III clasa a XI-a clasa a XII-a

Drăgoi Marius Motru I Stoian Bogdan Motru I Butaru Nicu CNT II Tigora Andrei CNT I Poşa Bogdan Motru II Goşea Ion Victor CN.Carol I Ciorobea Mihai CNT III Mitucă Atena CNT II Mema Alexandra CNT III Coandă Oana CNT III Nistor Ovidiu CNT III Prundeanu Andreea CNT III

Page 61: RMM 8.pdf

Concursuri

- 58 -

AMC 8 (noiembrie 2007)

Clasa a VI-a Clasa a VII-a Puican Tiberiu CNT Gold Driga Darie Vlad CNT Gold Mema Serban Costin CNT Silver Stuparu Denis CNT Silver Ciobanu Diana CNT Bronze Barbuti Alexandru CNT Silver Ciuciu Laurentiu CNT Bronze Zamfirescu Simona CNT Silver Barbulescu Ramona CNT Bronze Marghescu Andreea CNT Bronze Clasa a VIII-a Nicolae Andrei CNT Bronze Asproniu Robert CNT Gold Saceanu Andrei CNT Bronze Grosu Vlad CNT Gold Craciun Cosmin CNT Bronze Nistor Andreea Şcoala Gen.14 Gold Andreescu Madalina CNT Silver Clain Andrei CNT Bronze

AMC 10/12 (februarie 2008) Clasa a IX-a Clasa a X-a Agape Mihai Locul I Sosu Cristian Locul I Popa Andreea Locul II Nicoara Calin Locul II Cepesi Cristian Locul III Calinovici Paul Locul III Clasa a XI-a Clasa a XII-a Nica Flavius Locul I Hinoveanu Catalin Locul I Voicu Andreea Locul II Tigora Andrei Locul I Bechir Adriana Locul III Steanta Anamarina Locul II Nicolescu Andra Locul III Mariescu Radu Locul III Papala Diana Locul III

AMERICAN INVITATION MATHEMATICAL EXAMINATION Carapencea Constantin X CNT Coanda Oana XII CNT Croitoru Razvan X CNT Mariescu Radu XII CNT Tigora Andrei XII CNT Steanta Anamarina XII CNT

CIPHERING TIME TRIAL (Dec.2007)

Clasa a XII-a Clasa a X-a Clasa a XI-a Clasa a IX-a Coanda Oana Calinovici Paul Voicu Andreea Papa Florin Cainiceanu Andrei Sosu Cristian Nica Flavius Mirosu Raluca Tigora Andrei Marin Andreca Cristina Mema Alexandra Agape Mihai Hinoveanu Catalin Craciunescu Marian Daga Misu Suselea Robert Furcuta Ioana

PURPLE COMET - APRILIE 2008

(UNIVERSITY OF WISCONSIN-WHITEWATER) Mentiune - Echipaj clasa a VIII-a Mentiune - Echipaj liceu Mentiune - Echipaj liceu ¾ Asproniu Robert � Tigora Andrei 9 Voicu Andreea ¾ Andreescu Madalina � Coanda Oana 9 Hinoveanu Catalin ¾ Grosu Vlad � Carapencea Constantin 9 Suselea Robert ¾ Prunescu Flavius � Seitan Mihaela 9 Papa Florin ¾ Rachitan Laura � Croitoru Razvan 9 Agape Mihai ¾ Zugravu Rozalia � Tesila Bianca 9 Nicoara Calin

Page 62: RMM 8.pdf

Concursuri

- 59 -

H SSM

Concursul “ First in math “ prof.Irina Zaman, Colegiul National”Gh. Titeica” Dr.Tr.Severin

In vara anului 2007, prof. Irina Zaman , de la C.N. “Gh. Titeica”, a

participat la Cursurile de vara ale Universitatii Rutgers din New Jersey, SUA, obtinand o inscriere gratuita pt elevii sai, promotional, la Concursul American “First in math”. Elevii au primit un “user name” si o parola de acces pt pag:www.firstinmath.com

Acest concurs s-a desfasurat on line, in anul scolar 2007-2008 si elevii romani s-au clasat pe primele locuri, atat in topul liceelor cat si in top 100 –clasament individual. Acestea erau pozitiile elevilor romani pe 30 aprilie 2008 la finalul primului an: 19269 puncte Gheorghe Titeica, Drobeta Turnu Severin in top 100 V-VIII-Alexandru Florescu Gheorghe Titeica, Drobeta Turnu Severin - locul 4 in topul 100 al liceelor. 19269 Gheorghe Titeica, Drobeta Turnu Severin - in topul 100 al elevilor de cls a VIII-a –Alexandru Florescu

Trofeul primit de Alex prin posta arata ca este pe primul loc in Romania,pe locul 4 in lume la clasa sa si pe locul 20 in lume la clasele de gimnaziu.

S-a facut o premiere in ian 2008 dupa ce s-au primit din SUA medalii , diplome,jocuri, bratari, etc si au obtinut:

Florescu Alexandru-aur Fanea Alexandru-argint Mituletu Madalina-bronz

Pentru cei pasionati de concursuri online, anul 2008 a inregistrat o premiera in domeniu pentru Romania. Si anume, lansarea primului portal de concursuri educationale online din tara noastra, care poate fi accesat la adresa www.carepecare.ro. Startul a fost dat printr-un concurs de matematica cu continut de nivelul clasei a III-a, dar care se adreseaza copiilor cu varste cuprinse intre 8-12 ani. In cadrul aceluiasi portal urmeaza a fi lansate si alte concursuri, care sa acopere toate grupele de varsta, dar si cat mai multe discipline scolare. Cei mai buni participanti vor fi rasplatiti cu premii pe masura. Mult succes la www.carepecare.ro!

Page 63: RMM 8.pdf

REZOLVITORI

- 60 -

Tombola RMM Numele şi prenumele elevului ____________________________________

Şcoala_______________________________________ clasa ___________

Numele şi prenumele profesorului ________________________________

Decembrie 2008 @ S.C. ROCAST MEHEDINŢI S.R.L.

Colegiul National Traian clasa a V-a Damsescu Iulia; clasa a VIII-a Mituca Anda; clasa a XI-a Budirinca Alexandru, Buda Maria, Franculescu Diana, Cerga Alina, Zaharia Mihaela, Gridan Iulia, Ularu Ancuta, Tanasescu Vlad, Vilcu Constantin, Mutu Marie-Jeane, Nistor Ovidiu, Nica Flavius, Nicolescu Andra, Mema Alexandra, Picu Bianca, Stefanoiu Anca, Boboiceanu Silvia, Voicu Andreea, Crivineanu Florin, Vulcanescu Adela, Enache Radu, Bolbotina Alexandru clasa a XII-a Belbu Loredana, Porojan Otilia, Sandu Oana, Cotet Alexandra, Georgescu Alin, Saftoiu Mihai, Modalca Denisa, Gamala Andreea, Trasca Mihai, Steanta Anamaria, Paunescu Georgiana, Cristea Valentina, Buncianu Ilie, Ionica Adina, Marza Sebastian, Stavaru Mihai, Mituca Anda, Botea Silvia, Ganda Irina, Badea Sabin, Nitulescu Maria, Cretu Andrei, Tomescu Liana, Cucu Irina, Negrea Aurel, Raveanu Ioana, Mahut Alexandru, Pasat Cristina, Paunescu Adrian, Cainiceanu Andrei, Iordache Norica, Vladu Margareta, Bojinovici Sergiu, Hanes Anca

Scoala generală Nr.11 Prof. Victor Saceanu – Bestelei Ramona (10); Buica Ramona (10); Colta Ionut (10), Covrescu Laurentiu (10); Cutitoiu Elvis (10); Dumitrascu Ramona (10); Gavrila Costinel (10); Giubega Andreea (10); Gurgui Georgiana (10); Iancu Puiu Nicusor (10); Milotinov Narcis (10); Mitroi Ionut (10); Pacala Georgiana (10); Radut Nicusor (10); Stanchescu Daniel (10); Socate Georgiana (10). Începând cu acest număr al revistei, se vor acorda patru premii, prin tragere la sorţi, elevilor rezolvitori de probleme din revistă. Vor participa la tragerea la sorţi toţi elevii care au predat profesorului de la clasă problemele rezolvate (cel puţin una) şi talonul alăturat. Cele patru premii au valoarea de 50 lei.

Sponsorul concursului este S.C. ROCAST MEH EDINŢI S.R.L. reprezentată de dl. profesor Marcel Popescu

Page 64: RMM 8.pdf

COLABORATORI

H SSM

- 61 -

Colectivul de redacţie: Gheorghe Căiniceanu

Prajea Manuela Nănuţi Dan Dan Daniel Sitaru Dan Antonie Rodica Stretcu Daniel Ungureanu Octavian Nedeianu Dan Popescu Eleodor Chilea Ion Săceanu Victor Lupu Adrian

Decembrie 2008

Gheorghe Cainiceanu Grecu Vasile Trailescu Diana Giugiuc Leonard Elena Rimnicianu Victor Săceanu Osain Victoria Eleodor Popescu Dan Nedeianu Daniel Stretcu Ionică Constantin Gimoiu Iuliana Bondoc Gabriela Roxana Rodica Antonie Draga Tătucu Mariana Paponiu Dana Chilea Ion Lupu Adrian Manuela Prajea Raducan Emilia Daniel Sitaru Ştefan Marica Dan Nanuti Cristinel Mortici Chirfot Carmen – Victoriţa I rina Zaman Constantin Giugiuc elev. Nistor Adriana Mihaela elev. Ioana Ionas